Вы находитесь на странице: 1из 862
Contents 1 Putcatt Party 11 Review of derivative instruments 112 Galland put options 112 Combination of options 12 Pateall parity 121 Stock peal party 122 synthetic sock and Teasories, 123 synthe options 124 Exchange options 125. Curreney options Everlee Salton pmparing Options 21 Bounds for Option Prices 22. aly eeriseo! American options 23 Timetoexpry 241" Thee inequaes «. 242 Options in themoney Exercises Solutions |S Binomial Trees Stock, One Period 31 Riskeneutl pricing 132 Replicating portalo 33. Volt. Solitons 4 Binomial Tees—General 41. Mul-period binomial woes 42 American options 43. Curreney options 44 Futures, 43 Otheresets Bxereises Soltis 5 Risk Neural Pricing 511 Picng with Tre Probabilities 52 Rlk-Neutal Ping and Uilty rercies Solutions w conmENTS 6 Binomiat Tres: Miscellaneous Tops wz {1 Understanding early exres of options. ‘ 7 62. Lognormaltyand alternative tees 1 82:1 Lognormalty eee I es 622 Aemasve res 19 63 Banat olay = cca 130 Baercises i salons = : : eae 7 7 Modeling Stock Prices with the Lognorma Distsibuion 7A The normal and lognormal dvtbutons «+. 71 Thenormal distin : 712 Thelognormal distribution é 713 Jensen inequality 72. Thelognormaldntbution asa model or stock prices 721 Socks without dvgends 722 Stocks with dividends 723 “Confidence intervals" : 78. Conditional payos using the lognormal model eresee ae Solutions © 1 ing tock Prices toa Lognormal Disibution 181 2 Evdhngtheerphialy to Solitons ein 9 "The Black Scholes Formula ra 5.1 Black-Scholes Formula for common stockoptons ve S12. Black-Scholes forma or cureny options : : ” 93 Hack Scholes formula for eptons om ates ve rie : : Lue Saltins i 10 The Black Scholes Forme Gree 191 101 Grete im {Lt Beka : vse 1012 Gamma! a 1018 Veen z : 17 1014 Thee, 198 1013 ko ame 1018 Pa. ae 1017 Gromesses for portion ere 2s 102 Hay snd ltd concep ne 1021 Estey = ee peed aoe 1022 Related conopis See an 1023 Basti ofa porto an 10.3 What il Be teted ont ae Exereies ° 23 solutions : TEED ai ONTENTS: 11 The Black-Scholes Formula: Applications and Vola 111 Profediagrame before matty LLL Calloptions and bull spreads : 11.12 Calendar spreads 11.2 Voie : . 112.1 Implied volatity 11.22 Hlstrel volatility : series Solutions ae 12 Datta tedging 121 Overnight proton adel hedged portolio 122 The delta-gamme-theta approximation 123 Greeks for nomial ees Peee 124 Rbedging eee 125 Hedglag mutipe Gres ee 126 Whatil betestedont | sss vss ss Exercises : Solutions mo 13 Asan Barver, and Compound Options 131 Asan options 132 Barer options 133 Maxime and minima. 134 Compouncl options. 15.11 Compound option pity 1342 Amercan options n stocks with one discrete dividend 1343 Bermudan options : rerio vos : Solutions ae 14 Gap, Fachange, and Other Options 141 All-ornotlng options ea 142 Gapoptions ; 1421 Deion of gap options eee 14.22 Pricing gap options sing lack: Scholes é 1423 Delta hedging gap options 149 Fxchange options wns : 4 Other exatie options : : 114.41 Choose options. 1442 Forward tan options yereises Solutions oe 15 Monte Carlo Valuation 15.1 Introduction 152 Generating ogaotial random numbers =. 133 Simulating dervative instruments - 154 Control variate mathod : 155 Other variance reduction echniguee Brerises : ee escite teeen 2m S888888 ESSRSRRESESE SERRBRSRSS SRERRATES SEseE conrents Solutions 3m 16 Brownlan Motion a0 161 Brownian motion ean 162 Aitkmetie Brownian mation. ‘0 162 Geometcie Brownian motion. : aa Berises eran ‘ 85 Solutions a 17 iferentals 309 174 Ditfereniatng a ae 172 Thelanguage of Brownian mation 1 Solutions ates : 95 18 1stemma 399 rerlses cece a Soluons 2.020002 08 19 The Black-Scholes Equation a Sotudons 2.21. ae ce ena as 20 Sharpe Ratio 42a 201 Calulating and using the Sharpe tio cece Hh 202 Risk ree Portfolios 2 sotuons 222. 3 21 Risk Neutral Pricing and Proportional Portfolios an 211 Riskenoutl pricing “1 212 Proportional porto 43 verses M6 sohaons us 221 \aluing forward on * ea ‘ : ae) 222 Thelto process for" ea aa 223 Derivation af iy (S)wsg ue peeing Paes 45 22: Examples ofthe formula fr forwards on S* 455 eee ta 60 23 Stochantie Integration 07 21 Integration aan “67 23.2 Quadratievaiation : St 168 24.3 Diferenil equations : eee al Baereises cal Solutions 22. a a teste 24 nomial Tre Models for interest Rates 241 Binomial Tees 22 The black-Derman- Toy model 242.1 Constuetion of flack-Derman To binomial tice 2422 Pricing forwards and caps using 2 BDT cee Solon 25 The Black Formula for Bond Options 26 Equilibrium Interest Rate Models: Vasleek and Cox-ngers 251 Pricing Options with he Black Formula 252 Prcingeaps wits he Black forma Brerciss ‘Solutions 281 The impossibve mode! 282 Rguilbrim models 2821 Thetendleman Barter model 2622 The aseck model 2823 The Cox ngereal-Ross model 283 Characteristics of Vasiek and Cox Inger oss models 263.1 Risk-peatal pring 2532 ale, T}e®™" form DAA R(T} (Valen made) 25.14 Shaipe ratio questions 2535 o0/07 26.16 Yield on infield bonds 264 Deka hedging Solutions Practice Exams Practice Exam 1 Practice Exam 2 Practice Bxam'3 Practice Exam 4 Practice Exam s Practice Exam 6 Practice Exam 7 Practice Exam 8 Practice Exam 9 aces so. ot 10 Practice Fxam 10 1 Practice Exam 11 Appendices [A Solulonsfor the Practice Exams Solutions forPeatice Exam Pees . Solitons fr Practice Exam 2 Satins fr Practice Exam 3 Solitons for Practice Exam 4 Solutions for Practice Exam 5 Solitons for Pract Exam & Saline fr Practice Exam 7 Solitons for Practice Exam 8 Solutions for Practice Exam 9 Solution for Preece Exam 10 ‘Solution for Practice Exam 11 Solutions to Ola Fxams [LL Solos SOA Exam MEE, Spring 2007 182 Soltis to CAS Exam ,Sping2007 =. 1B Solulns to CAS Bua 3, Fall 2007 : 14 Soltione to Gram MPE/3 Spring 2008 [BS Soludans o Sample Questions Lessons Corresponding to Questions on Released and Prats Exams Standard Normal Distribution Function Table Preface Welcome tothe MFE/3F exam! ‘The SOA is developing a new Risk Management cedental, CERA.Stadents can qualify for this creden tial without staying Life Contingencies. To make thi posible, tis exam wat spl of fom Exam MIC in Spring 2007 The CAS decided to jon the SOA an olny Sponsor his exam bening fn Spring 208, ‘You willbe studying, almost exlusivey option pricing There wil be shor discussions of forwards here and there, but almostall ofthe de you wl be pricing calls ad pts You may have encountered asi norman ‘on these when you took Exam 8/2, bt you dont need eveything you lerbed in that course. Abit summary ‘of theinformation you needisgivenat the beginlngar Lesson 1. The restolthe course can then be dived i ‘the elmentarypart~Chapters 914 and 18-19 fom the textbook, covered in lessons I-15; nd the advanced ‘part—Chaptes 20-24 fom the testboo, covered in Lessons 16-25. The elementary par oes the flowing order 1. Principles relating pices ocallsnd pus, and relating pices of options o each ater and bounding thet, ‘These principles donot develop exact pices for options, but are general and easy to derive 2, Valuing opcons using binomial ees, 3. Valuing opons using analyte methods (lack Scholes. 4. Doinion ofexoe options, and pricing methods where avalable 5. Thelognormal modal for stocks 6. Monte Cal valuation, ‘The advanced pat covers: 1. Theory behind valuation formulas: Brownlan motion, lems 2. Valuing options on bones interest ate modes. ‘The MeDona textbook ian easy read forthe elomencary part, but becomes more difcltfor the advanced prt I est avoid higher mathemas. Asa esl willoften skp steps int decvation of formulas Iealso testo develop diferent formula null, which doesnt always make them eas toundertand found the ‘material in Chapters 20-23 (wher to proceses ae discussed) an the beginning of Chapter 24 wer nest rate models are dscused) particulary eificat. These chapers ae extensively covered in Lessons 16-29 and ‘Many ofthe textbook endo chapter problems ae wort looking at. Howeve, many depend on spread: sheets provided with the textbook. Onan exam, you wil nether be given those spreadsheets nor allowed use Bice, you us the textbook, refer othe following website fr erat forthe fst printing: beep: / aww ettogg. northnes ern odu/faculty/edonsid/sta/typoe29_01.Atal or for ecratafor the socond printing (which includes ational erat forthe fs printing dneep://iw, kel ogg. norvhgestern.odu/faculty/acdonad/bts/typos2e,02. heal ‘Table 1 shows the welghts given wo diferent topics on this exam inthe CAS syllabus and on the SOA sam ple an he released Spring 2007 and Spring 2009 exams. The non-sylabus question i on Perpetual Options. & PREBICE “Table 1: Distribution of exam questions Namberofauestions os Sar Tentont | nua! [png [TP Sane] Sonne |S Tope Chaps [isons | 200" | zor | quetons | sur” | Eon veweatpiynde | 9 laa] a|sl s [a] 2 | ed ert 2.dinoma es wu fos | afal ae [a] s 3. Lognormal model 18 cy o o 3 o o] ‘hak Sele wv fen} ofa] a fal 3 5D dng we fie] afta] s fala 6. resins uw fou] i fa] oa | a | 2 TMoneaowuain | ts | is | o | o| 's | o| o topcase wa fwn| i }ol a | 2 | 6 Sineearemoats | at” | zen] 2] o| 8 | 3] 3 Noronytabue Loteo| «tila eal Gea Sea apa Material in Lessons 7-8 and Lesson 15 was added tothe syllabus forthe Fal 209 sting and therfore does not appear on thecalereleased exams. According the bus, 65-75% ofthe exam willbe on tpl 12:8,and 8 of able I: 5-10%an ope 10-15% on ople7 and 10-15% on pies (Curent the ME/S¥ exam is 23-hour exam and shat 90 questions, incadng plot questions, On com: puterbased tests pilot questions are questions that are not graded. Instead, thee dfculy is assessed based ‘on how well stdens do on them, so hat the pass mak on exams wih hose quenons consent with other ‘administations However, the exam was about 20 questions before Fall 203, and wes about 2 questions fom Fall 2009 nt Fall 2020. Wit gard to the SOA sample questions, note that questions 1-16 were not on any exam. However, some ‘of questions 17-30 and some of the later ones, according to stodent reports, were on unreleased exams. 1) patcular, 19 and #22 vere on te Fall 2007 exam (and possibly oder questions inthe #17420 range), wile 12), #23, 425-09, and #47 were on the Spring 2008 exam. Also, question #48 was onthe Fall 2008 exan, and ‘questions #50-¥82 were ken from the Spring 2007/4 Exam, and question #61 is sad to have been on the "2009 exam, Many ofthe questions ater #64 also are similar to student reports rom unceleased ears While ‘the sample questions largely come fom pas exams, they are probably the harder questions fom hose exam ‘the overall dificult ofthe real exam sess than that ofthe sample questions. While te el exam puts fally Ineay weight on tb processes, my gues Is that you should expec questions (or about 20% of the exam) on tp processes, nt the rato implied bythe sample question dstibuson. The dstbution ofthe Spring 2000 ‘xa, adjusted forthe adltion of new topes, is probaly more nda of the distribution of exam questions. ‘he CAS exams virally skipped Brownian motion and interes rate models, and the fer questions they posed on these topes on their Spring 2007 exam seem to indieat lack of experts In thelr eam commie. The disbutio of topics and dificltyof questions onthe CAS exams not ndtv of tae exams. os Presace a This manual This manual gvesyou compete coverage ofalsyllabustoples.T help youcheckhow mich you ae sbsorlng, ‘thor are quizes wih euch lesan; these are sraightforunrd exercises which you should work out at you get to them. Solutions to quizes are atthe end ofthe leon, after the sluion to the exercises, The execlees a the en of each lesson are designed tobe exaike (although sometimes they ae abit ong fran exam), ‘equing only eaeulatorand. abe ofthe normal disebution to solve. Working these out wll help you learn the concepts. "Natethe lowing valuable featres at heen ofthe anual: + Solutions o relevant questions rom all leased exams: he Spring 2007 and Spring 2009 Exams MEE and ‘the Spring 2007 an Fall 2007 CAS Exam 3 + Solutions to the 76 sample questions. The solutions provided by the SOA ar longer and often include ‘tional commentary and educational material My solution gett the point and are meant to ndate ‘he method yout use onan exam + Across reference indicating the lesson covering every relevant question fom the release exams andthe sample quessons and asl crss-eferenceforalqestons inthe twelve practice exams. + Anindex Anote on notation: the MeDonald textbook uses N(x] indicate the cumulative standard normal distrib lion at. Many other textbooks use Mfr the same concept, andi fat Tuse (2) inthe Exam MLC 3, and (C/4 manuals: However, fortis course, Ifllow McDonald and nancial economies tration and se Ns). New for this edition ‘Siificant now features inthis edition a= 1, Allcalculations involving the normal distribution have been redone in accordance with the precision of the neve Prometric eal, 2, 5 new eereises on various topes, neuding 1 on interest rate models, were added, 23, Lesson 26 was organize, 4, Duplicate and detective practice exam questions wore replaced The normal distribution table ormulsin this course use the normal disbution. Most students wil be aking this exam a Prometze se, Prometric provides standard noma disrbtion calalton. See htp://ww.pronetric.coa/SOh/MFESF calculator hem to see how this works, The calculator provides values af the cumulative noel distribution function and ts inverse to deta places "when working problems in this manual, you may ces HORASDIST and NORNSTNV functions to perform normal distribution eaeulatons in a manne similar othe way yu wil do them on an exam. Arora it bation table is provided in Appendix D, You cat use thi ina pinch if you dont have access (oa caleuatr oF rogram tat can ealalate norma distribution vals, Tnadditon, you willbe given formula sheet. See x -ntep://wwe,promatric.coa/SOA/MFESF_calculetor.nex {or tis sheet. 1 provides the standard normal density funtion, dhe lognormal density function, and moments olthelognormaldxsibtion, you are aking the exam at a paperand-pene se, you wil be given formula sheet and «cumulative normal distribution able, Cacenthy the ul for this tables hep: //aw.s0a.org/tilea/pat/odu-2009-fa11-nfe-table.pat ‘The formula shoe has es for the use ofthe abl, aden hat You should not interpolate inthe abe se ound vals Rea those ales for more deals ‘Since moe students il be taking the exam under CBT, the man uses the more exact method of elu lating normal distribution values, namely pace precision, Errata Please eport al errors you find in these notes to the author. You may send thom to the publisher at ma2.2 studymanuals.com or drectyo meat erratadaceyourexans.net. Please ideal he manual and eon {he ezor sin. This the ninth eiion of the Fxam M/F mans “Aneraalistwilbe posted at errata. aceyourexans net. Check this eras frequent Acknowledgements "woul keto mank he SUA and CAS tor allowing me to use questions Hom ther ld exams, The creators of FX, BF, adits mullite of packages ll deserve thanks for making possible the profes: sonal ypesting of hismathematical materia. Tish acknowledge all tudents who sen in erat or recommendations for improvements, whether many or few. Special thanks to Jeff Raven fr his mathematical sighs. "paral at of other suerte wt pointed ot exons Mak Aan Ryan Amma, El Barich, Ease Bal Be ‘any Barer, Andie Tl Eu Shen Berman, El Bochner Nichols Brae. Kon Burt, Yang Can Mell Car Jet Cet Garand Can Grace Chang, Aaron Chas. Biy Cen, Fling Chong Ae Cho an Chung, Sve Cla, Douglas Cat, Su Cts: Thomas Dat Cameron Daniel aie Oui, Lindby Daniels Magdalena Died atk Di flit near Dana Dor Bryan Donker, Ades x Peso Eon fal, Mather, ‘cheved pra, Matthew Ets, jeremy Evans, oe Faure, tons Frog Amara Felner, Yucog Fe Dane Fer ander Flinger ce Fr. ce Ga Georges Ghana, Aryeh Gch, rt ibn, Micha Golsen, Tovah Gracy Tvs Greeley Gay Aison Gut, Dvd Hamann, Aaron Hahn, Seat Handley. Fyn es, Witlam Henderson. enn Ho, Spmphone Home, Susen Home, Steps, Set Humes, Cala Hvar, Doyen sng tie ler lng, Dee nse, Mate enc, on lag Sewn Jones yon ang, Derek kash "rank Ky Prec Koppelman, Rela Rtste Ante Kyo, ako Ranazaa, Deke Kung Jacqtne al, Sex [stone Gry Laan, uc! Lata An Le, Domini Le Jneph Seung re Yok Le, Yong chor Leas Lng, ‘Chass Lindberg David Li, vy La, son Mack Wison Ma Ces Mave, Cpe Macc ron Masri ‘com Ras Mawk ein Mebith, Stat Mcroden ate Meow linny bce, hip Mekeran sep Malt, Pu Mollena Lag Muti Csiopher Nes Eat Nas, Bet Ne Nb. ome Oka, Ar Oars Pte ‘George a xn Peas, Stang Pg Cole Se Peter Kn Ming hon. Amana opbam jun Pb oa Pre, Tad emits lara Ryne, Va Sura Dai Schenck, Dem Schobel Simon Ser rn Shige: David Sune, Ch Ses Ca ‘Sion, Core spas, Ro teat, Alon Stoop. fseSultg, cols Sundsad me Swanson, Susan Spans hoon Te, Georg nnn Suan Thoma Nan lan Shel Tula, Teens Ta Mayer Tolan Leon hah ‘oppor Sam Teng aul Uh, Vege, Edward Wang Kae Hen Wang, Cars Washou Leighton Nese, Cet ‘Wal Kt Vilma, amie Wong, ctor Wong, Wood Karla Wysal Ere Yes, ode a, Bai ance Dee “nein, sion Yuen, Ben Zag, Aron Zeige, Cheng Zang. Soy Zang "ial ike othank Presi Kzysa Ostasrwsi fr pling oan er Lesson 1 Put-Call Parity Reading: Derivatives Markets9.1-9.2 ‘expect one ot more exam questions base on hisleson, 1.1 Review of derivative instruments ‘This review section will not beet tested on. Ils background mater fom Exam F/2 that you are ex pected to already know Ifyou donot wish to eew, proce immediately to Section 1.2, page 10. 1.1.1 Forwards ‘forward ian agreement o buy something a future date for a certain price, We wll se the notation Ft Indeate the price tobe pad atime T na forward agreement made atime ¢ to buy aa em tte 7. Notice ‘hat no payments made at time ftheonly payment made Isa time T. Ifyou purchase forward on stock at time f, you wil pay atime Pand ect the stock. In pretice, there my be no physical transfer ofthe stock. tat, stent nye weve by yt receiving the difeencea the pica the stockat time Tand Fi Ifwe tS be the pic ol te sock at ie then you would receive Sr ~ Fy which may be postive or negative. Ths he payois linear function ofthe ‘value ofthe stock at me 7. Suppose we assume Fir = 0, Ife stock price atime T Is 30 you then pay 10, TF the tock price a time 750, you then receive 10 The paso atime T asa funeson ofthe stock price atime T.Spisshown in Figure 11 ‘forward agreement sa customized comet. tures contac, n contrast, fea standardized exchange traded contract whch is smart a forward in that isan agreement op acertan amount atime Tora ‘certain ase. However, atures contac s marked o market dala pal of ts value is made berween the parts each day. There are oer differences between forwards and futures. Howeves, inthis cours where We ‘ve only interested pricing ther, we wil not dietenite between them. Payoh 0 2a] | a ee eee Figure 1.1: Payot on 2 forward having price 40. The payoie Sr~ Fir, where Fr = 40 Nesey etn 1 Speoreaae 1. PUECALL PARITY Let us determine Fz. As we do so, we shall introduce essumpdons and terminology sod throughout the cours. One assumpdon we make throughout the cours is that there rikfee interest paying investment fan invesoment which wll never default. you invest X in this investment, you are sure wo recave K(L+1) at ‘he end ofa yar, where is some interes rte We wl always express interest a a continvosy compounded ‘at instead of as an annual effective rate, unless we seyatirwase. We will use the ltr or thi at. In Financial Mathematics the leer 6 i used for ths concept, but we wl be using 6 fora diferent (although ‘elated concep, Ar you Teared in Financial Mathematics +e". Soifthe ike effective annual etn ‘sas we woul say that r =n 1.05 = 0.0479 rather than OS. In elif, Treasury secures play the role of ikffe interest paying investment Here are the assumptions we make: 1, Ieispossibteto borrow orlend any amount of money at he risk-free rat, 2, There ate no transaction charges or nes 8. Arbiragesimpossibie ‘An Aritragesa set of wansttions which when combined have no cost no poss oflss and at east some possibly ifmo certain) of pro In order to make arbitrage Impossible, there are two sets of transactions Teslng the same end rest they mist both have the same pie since othervse ys coud enerimocane set of wansactions asa solr andthe other set as buyer and be assured a profit with no poss of ss Forwards on stock ‘We wi consider posses ror me stack: 2, The stock pays no dividends 2, The stork pays disrete dividends 8. Thestock pay continuous dividends Forwards onnondlvidend payigstock ve begin with forward ona nondividend paying stock. The onan agreement entered atime O and provides for transfering the sock at time T. Let, be the price ofthe stock time f. Thee ate two ways you can gum the stock ime 7 ‘Two Methods for Owning Non-Dividend Paying Stock at Time T Method, Ruy stocka ime 0] Method 7: Ray fora on woe ‘and ha ioe T ‘ttime and hold ito time T Payment ine O Se 7 Payentatiime 1 a Tir -ythe principle of no aria, the rwo ways mst have the same cos. The cosa tie O ofthe ist way is ‘othe price of the stockat cine, The cost othe second waylapayientof Fy at ne Since youcan Invest at the isk reat, every dela invested at ine O becomes e”” dollars at ine T. Youn pay fy at time Tby Investing Fre"? atime 0. We thus have Brea! _Wehave priced his forward agreement. Ld, REVIEW OF DERIVATIVE INSTRUMENTS a Forwards onastockwith discretedividends Now let price a forward on advdend paying stock This means Iepaysinown cash amounts at known times There are the sametwo ways to own aock atime T: ‘Two Methods for Owning Dividend Paying tock at Time T “athod Ws Hey sackat time 0 Method ¥: Rayos on oer and hold it totme atte Dd hold ittodime 7 Ppmearaeine o s o 7] Paymentattime 7 2 cs However you use the fist vay, you wil aso have the stock’ dividends accurate with interest which you wont have with the second way: Therefore, the price of the second way a of ime T mist be cheaper than the price ofthe ist way aso ime Tby the accumulated value ofthe dvigends. Equating the? costs andleting ‘Cunalue(Di be te accumulated value at ne Ta dividends from te Oto time T, we have fr ve"7 Oumahu(Di») an Forwards on stock index with continuous dividends We wll now consider an asset that pays dvdends at ‘continuously compounded rate which ge reinvested in te att. In er words rather than being pd os «ash dividends at certain times the dividends get envesein thease continously ott te lnvstr ends ‘Up with adalional shares ofthe asset rather than cash dividends. The continuously compounded dividend rate willbe denoted 5 ‘This could bea model for astockindex stock index consists of many Rocks paying dividends various ‘umes Ase simplifcauon, weassume thew dividends ae unifor, We wlloften model single stork not jus a tok index, fi had continuous reinvested dividends, using 6 instead of epic dividends Lets price forward on astock index paying aiden at ate of, Kyou buy the stackindex at ime and ‘pays continuous dividends atthe rated, you wil have 2°” shares ofthe stckindexat me 7. Torepleate this with Forwards, since the forward pays mo dvidends you wold noed to ener ora agreement to purchase 1257 shates of heindex. So here are two ways to ow shares ofthe stock index at time ‘Two Methods for Owning o*” Shares of Continuous Dividend Paying Stock Index at Tne 7 athod 0: Ruy sock index at | Method #2: Baye” fords on time Gandholditionme T | stockinderat time Dand hold t0 time T Papmentartine 0 = 7 Paymentat tine 7 o ra Thus to equate the two way, you should buy 2” uns ofthe forward at time 0. Aime The accumulated «cost of buying the stock index at med 6 Se" wile the cos oe” uns ofthe formal ageement Fare", cowehave Bre Bra syelO 02) ‘We ee here that andr work in opposite directions. We wil inn goneral that tends to workike a negate a bens 1. PUTcALL PARITY (Quiet *Forasioak you are given © pays quarry avidends F020, (i) tehas st paid a avidend (Gs prices 0. (i) Theeontinoously compounded risk-free interest ati S%, Calculate he forwanl pric for an agreement ro delve 100 shares ofthe stock sik months from nove "forward on a commodity works much ike forward an a nondvidend paying tock. Wf dhe commodity produces income for example you can eset ou, the income play the rol of vdend: ie eommodiy roquires expenses for eample, storage expenses), thes expenses Workke a negative dividend Forwards on currency For forwards on cureney, tis assumed that each currency hs sown rte interes rate. The ike terest ate forthe foreign curren play the role ofa continuously compounded dividend ona stock Tor cxample, suppose a forward agreement at ine 0 provides forthe delivery af eos for dln time: ‘Let be the skew ate in dollars ad re the skeet In euros. Let xy be the rate atime. The two ‘waysto ave euro aime Tae 1. Buy e-7 eurorat ge O and et them accumulate toe F. 2, Buy forward for | euoat time 7, witha payment of Fr atime 7 “Te mrs way coss ze dolar at ume o-The second way can be funded in doa ar ume Uby puting aside fe Fr dallas ate 0. Thus eM Reane Be ‘Quie 1-2 A yen-denominated forward agreement provides forthe delve oF S100 at te end ofS months The continuously eompounde rik-re rie for dolla s 5%, andthe continuously compounded sk fee at or yen s2% The curent exchange rates 1104/. Calculate the forward pice in yen or this agooment. ‘Asummaryofthe forward formulas sin Table. Thistable makes the easy generalization fomagiements ‘entered atime 0 agreements entered at timer Ieisalso possible to have forwards on bonds, and bond interest would play the same role as stock idends ‘nuh forward More typical are forward rate agreements which guarantee an interest rate 1.1.2. Call and put options ‘Aca option permits but doesnot requ, the purchaser o pay a specfic amount K a time Tin etuen for an aise. ha cal option, there ae two cath flaws payment ofthe va ofthe call option is made atime and ‘possible payment of Kn ern fran assets made atime 7. The payment ane , which the rei ofthe call denoted by CURT)? Atte T.rathe than actly paying K and receiving the asset a monetary Lanes si crc Nan hee om at ey ‘thts team am ots est pa bhai puns ogrtenr eee Li, REVIEW OF DERIVATIVE INSTRUMENTS 5 “able 1.1: Forward prices 2 time 1 for settlement a time T Undesving see Forward price "Non-vidend paying stock ser Dividend paying stock 50-9 Gamat) Siodkindex sie" ‘Gunency denominated in corency a | yg Fordativery ofcurency f i 20 0) ° ~ct00.7) — a se a — oe a (aor ett! ise onl open (0 Pst vmeton puter Figure 1.2: Pay net of initial investment (ignoring interest) on option with 2 sre price of 40 settlement ofthe diference usually made. Sy ithe vale of the asset a time T, te payottisman0,S;—K) The one who sel the call option scaled the rte of the call option "Aputoptin sthe counterpat toacal option. lpr, at doesnot equ, the purchaser to sell an asset ate Tfora price of K. Ths there ta payment at ine 0 ofthe premium ofthe put option and a possible ‘eceiptof Kinet forth asset at time F The payment atime 0, which the premium othe pus denoted {KT} The monetary setiementat time T ifthe assets then worth is aK, KS) ‘ctscaled the sre ple, gute 1.2 shows the net payoff of call ora put, assuming the srk pice is 40, The cost ofthe option tudded othe pay at theme 7 interests ignored although we wont gore when we price the option, ‘The options we have described are European options Buropean options can oniybeexeceed atime 7, nat before In contst Amerzanopionsaliow the option toe eerie at any ie up one 1.1.3 Combinations of options ‘Various strategie involving buying or selling moe than one option areposibe, These are discussed in chapes'3 ofthe MeDonald textbook ad you lear about them In Course FM/2. Is probably not necessary to Know aout them for his exam, since exam questions about them wil deine ther for you. Nevertheless, ete 3 Shor summary of them ‘When we buy X we ae saldo be long X, and when wel X, we are sid tbe short X: Long and short may he used as adjectives or verb. 1. PUT CALL PagETY Option strategies involving two options may involve buying an option and sling another option of the ‘ame kind (both alls or oth puts, Buying an option of one kid and selling one ofthe other Kind, or ying twooptions of diferent kinds (which would be sling two optians af diferent kinds from thes parspecte). We ‘wallbe assuming European option fo simply. ‘Spreods: buying on option ond seling another option of the same kind ‘An example ofa spread Is buying an opton with one strike pric and sling an option with diferent strike price Such spread is designed to pay offfthe stock moves none dection, but subject oat Ballspreads A bullpen pays ofthe stock moves up in price but subject to sit To create bull spread with als buy aK stk allan sella Kath call > Ky. Then at expiry time T, 1. iS & neither option pays 2. IK; <7 Ky, thelomer-strike option pays Sr~ Ki, whichis the net pay 3. MSp> Ky thelower-stke option paysy~K; and thehigher-suhe option pays K;, So thenet payott {athe diterence on KK. ‘To create bul spread wih pus, buy aK) strike pt and ella Ke-strike put, Ks > Kj. Them at expiry te 7, 1, IS ©, helower strike option pays K,~ Sr and the higher strike option pays K Sy. foranet pay of Keo, 2-18 2Sy5 Ky th were option is worthless andthe higher strike option pays Sto he net payors Sy ~ Ki. 8. Sr» Ks both options are worthless, ‘Although all he pays othe purchaser of bl spread wit pts are non-positive, they increase with ncreas- ingstckprice. Since the posion has negative cos (he option you bought cheaper than the option yousol), youl gun as lng asthe absolute vale ofthe net pays lower han the net amount recived at inception, And the absolute vale ofthe nee pay decreases as the stock pice Increases. ‘Adlagram ofthe pet profit on all spread isshown in igre I. ‘Bear spreads how spread pas off the stock price moves down in pice, ut subject oat. Toceste a bear spread with pus buy a Ke-srke put and sella K-strke put, Ky >. Then 1, Sy K; neler option pas 2, IK, > Sy 2 Khe higher strike option pays Ke 43. IfSy- Ki, Then 1. IS, 2 Ky, the higher strike option pays ~ Ke and the owersrik option pays Sy— Korat payotl oki Keo 2. If > Sp > Ky the higherstike option is worthless andthe lowersrike option pays Sy ~ Kj for amet payotfof Ks -S1<0. 5. IS Kj) and buying 2 bear spread of puts wth strikes Ky and Ry. This mens buying ad eling the following Bal Bear suite | spread spread %; | Buyeal separ | seteat_suypue Assuming European options, whenever you buy acall and sll a put at the seme strike price exercise by one ‘ofthe parties is certln (Unless the sock price & the strike pice a matursy in which eae bot optlons ate ‘worthless sos equlsalent oa forward. Thus a box spread consists ofan agreement to buy the sock for {and slit for K, with definite prot Ky ~ Ky. If priced eoeecty there wil be no ga o lose regardless of the ‘Stok peice at mati ‘Buttertyspreads A butterfly spreadisaree-option strategy all options ofthe samerype, consisting of buying ‘bull spreads with sce prices Ky and K; > Ky and sling m bull spreads with sik pices Ke and Ks > Ke, ‘with m and selected so that or eal ithe stock price Se expt greatr than X, the pays net to zero. (tps are sed, range eso thatthe pays are ze iS; < Ks) ‘ets workout the relationship beeween mand fora butt spread with all + For the. bull spreads which you buy with strikes Ke and Ki, the payo is Ke Ky when the expiry stock prices Ks > Ke> Ki 1. PUTCALL PARITY Net prot 20 10} ——————————— 0 w oe 70S Figure 1.4: Net profit on butely spend. European cl options with strikes 4, 50, and 60 are use + For them bull spreads which yu sll with strikes Ky and K, the pao Kj K, when the expr stock pees Ky > Adang up the pays UK Ki)= mK = Ki) ns — Ky) = mC) Inthe simplest caso, m = and Kis hal way between Ki and Ky this i a symmetric butter spread. Otherwise he buttery spreads aymmete ‘Tae payoonabuterty spreadis0 when >< K,Ithen increases with lope nuntlitreachesitsmaxmum ats =, at hich point ic np ~K)}- then decreases with slope mn reaches Oat Sy = Kya 60 focansy> Ks diagram ofthe net profits in Figure 1 There sno free lneh, so the options involved must be priced In such away tate buterl spread loses money the Hal tock price below Kyo above Ki. This implies that option prices as faction of stk prices must be conver, This wl be discussed on page 40 Calendar spreads Calendar spreads involve buying and selling options of the samme kind wish diferent expty dates. They wl be discussed n Subsection 1112 Colles: buying one option and seling an option ofthe other kind Ina cola, youll cal with ste K; and buy apt with ste; Ky; then this strategy alle a rangle Diggrams of the net profit ofa straddle an astrangle are shown n Figure 1.6. ‘Were done with review. 1.2. Put-calll parity Inthis lesson andthe new, ater than presenting a model for socks or other assets So that we ean price op- tions, we discuss general properties that are ue regards of mode. In thisesson, we dscus the relationship ‘benween the premium of cll and the premium ofa put. Suppose you bought a European ell opdon and sold «European put option, both having the same under Jyingasse, the same sre, andthe same time eo expiry. In shisenire section, we wil dal onl with European ‘opto, not American ones, so Nencelorth “Furopeat” shouldbe understood. above let he vale ofthe Geryingasset be at timer You would chen pay C{X. T) ~ P(X, T)at ned. Interestingly, an equivalent est fan be achieved withou using options at al Do you se how ‘The pointsthatat time T-oneof the tw options is sure tobe exercised une the price of thease at time ‘happens to exactly equal the sukeprlcetS, =X), 1n which case hath opionsare wortless-Whichever option Iserercised, youpay K and receive the underlying asset: + 11S > K; you exercise dhe cll option you bought You pay K and ceive te asst. + 14> Sythe counterparty excels the pur opin yousold, You reeive the asset and pay K: +15; =K- ledoesnt mavier whether you have Kor thundering asset, ‘Therefore there are wo way to receive Sp atime T 1, Buy aca option and sella pt option at ime 0, and pay at me 7 2, Entra forward agreement to buy Sand ate 7 pay the pie of the forward agreement. ‘By the no-arbitrage principle, these two ways must cost the same. Discounting fo time 0, ths means CURT) PK TH Ket Fre Par Cal Pay] 1) PRT) y—K) as) Here’ another way to delve the equation, Suppose you would ike to have the maximum of Sy and K at ‘sme T- What ean you buy now that wiles ing this maximum? There are two chokes + You can buyatime-7forwatd on the asst, and buy a put option with expr and strike price K. The forwad pices but since you pay a time, you pay e°" irfr the forward + You can buy rss investment maturing for K time, and buy a eal option on the ast wlth cexpty T and sie price K- The cost ofthe sfc investments Ke“ Both methods must have the same eos 50 Mg AKT) = RETEST) which is the same as equation (3. ‘The Put-Call Paty equation ets you pice a pat once you know the price ofall a go through specie examples 1.2.1 Stock put-call parity Fora nondividend paying stoc, the orward price is Far = Sv", Equation (1.3) becomes cuK.T)= PK.) ‘Te righthand side isthe present value ofthe asset minas the present value ofthe strike ‘Buspi4 1A Anondividend paying stock hs a price of 40, A European calloption allows buying the stock for Sat the end of9 months. The eotinvossy compounded risk-free eS. The premium ofthe ell option ‘520, Deicrmine the premium ofa European put option allowing sling the ck for 45 atthe end of 8 months. Ker Answer: Dont forget that 5% 8 continuous compounded rte, UK, 7) PK. T)=S— Ke 294-PIK,T)=40~45¢"8280 28 PUK.) a0 (k,n) =204+9.0075 [STENTS] a ‘Aconvenlent concep s the prepaid forwed. This the same asa regular forward, excep tha the payment ‘sade at he time the agreement is entre, time r= 0, ater than atime T. We use the notation f for ‘the valueat time # ofa prepaid forward setting atime T. Then Fey = e-"" Fs. Mote general ithe forward ‘maturing al ie T x prep at cme ¢, Ff e-"- , we can translate the formulas in Table Ino ‘the ones in Table 12, In this able Pv; +i the preset vale atime fof payment a tne 7. Using pea “Table 1.2: Propid Forsard Prices time # for stlement at time Undesving ane Forward pice | Prepaid forward price ‘Non-dividend paying tock aad Ss | Dividend paying sock 5.09 —CumialuetDn) | §,—P¥, (DH) ‘Stockindex 3a seer ‘Gurency denominated in eanreney @ | gomntt-n neni for dairy ofcurency J forwards the putea parity formula Becomes ao ‘Using this et dscussadlvidendpayingstock. Ia stock pays diserete dividends, the frmula becomes (UK, T)~ PUK, T) 85 ~PVas(Di08)— Ke"? a9 [BxqnPUs IB A stock price 45. Th stock will pay a dividend of fer? months, A Buropesn put option with sstrke of 42 ad an expiry date of 3 months has a premium of 2.7. The eantinuousy compounded risk-free tele, Determine the premium ofa Buropean cll option on the stock wih che same stile and expiry. 2 41, PUECALL PARITY Answers Using equation (1.5), UK. T= PUK, T)=S.~ Pia Des)— Ker"? (Dis) the present value of dividends ithe present valu ofthe dvidend of iscounted 2 months at 5%, x(t €(32,025)~2.71 45 (18 —aze*H0= = 45 0.991701 —42(0987578)=2.5500 ‘Quiz1-3 A stocks piceis 0. The stock wil pay a dividend of after 4 months. A Buropean call option witha Strike of50 and an expiry data months haa premium of 1.2. The continously compounded sk ree ate isa Determine the premium of European put option on the stock withthe same strike and expe. Now let consider a stock with cntinuows dividends at ate 6, Using prepald forwards, put-call party be UK.) PUR.) = Sy0°* Ke as Bxwwrur 1G Yow ate gens (0 stocks pees 0. (i) The continuously compounded risk-free rate. (ut) The stocks continuous dividend rte is 2. ‘A Buropean 1-year all option wit strike of 50 coss 2.34 Determine the premium fora European 1-year pu option with astrke of 50, Avswur: Using equation (8) ‘40 9801967) ~0(0921163)=-6 91787 se+aoc7er=[Baaa7] o (ule 4 You are given (0 Astoeks prices 57 {G) The continuously compounded rsk-e rate 5%, (i) Th stocks continuous dividend rates 3%. A Furopean month putopson witha srkeof 8 costs 4.46, Determine the premium of Buropean month ell option with strike of 5, 12, PUTCALL PARTY 13 1.22 Synthetic stocks and Treasuries ‘Since the put-call parity equation includes terms for stock) and cash (A), we can rete asyathetic sock with ‘an appropriate combination of options and nding With eandinuous dividend the formal is CUR, TI-PK,T)Sye#— Keo Som (CUR T)—PK. T+ Kerr) an For example, suppose the isk-roe rte S%. We want octets an investment equivalent to a stock with ‘eontinoous dividend rat of 2%. We can uses srk price ni any expiry let say 4D and I yest, We have = (C(0.1)~ a0, 1}+ 406-5) tse ‘So we buy e®% = 1.02020 call options and sell 1.02020 put options and buy a Teasury for 288178. Ache fend of a year, the Treasury wil be worth 28817%e! = 4.08. An option wil be exercise, so we wll pay 40(1.02020) = 40.308 and got 1.02020 shares ofthe stock, which is equivalent to buying I share of he sfck ‘orginally and reinvesting the dividends. alsidends ar discrete then they ar assumed 1o be fied in advance, andthe formula becomes (K,T)~ PUK, T)=$5—PMedends)— Re-"™ i= C(K,T} PUK, T+ PUlvidends) + Ke For example, suppose the risk fe ate 5% the tock is 40, and the period | yes. The dividends ae 05 saplece at theendof3 months andthe end of month, Then thee present val is os To creaea symtetistoek, we buy eal sella pt anion 0.975994 40e-" ~39 0246. the end ofthe yeas, well have 40 plus the aecurmulated valucaf the dvidends. One ofthe options wl be exercised soe 0 wl be ‘exchanged fo on share a the stock. "Tocteate a synthetic Treasury, we earange the equation as flows UK.) PeKT as) ‘We buy e- shares of the stock and spt option and sla cll option. Using. 2 year 6 maturity agin the total cost of thle is Ke~"™ — 400° stock fr 40 (sine one option willbe exercised). This x equivalent o investing na one-year Testy bl With ‘matuty value 4, flvidends are sree, then they ae assumed tbe fixed in atvance and canbe combined with the tke ee a oliows: Ke-"? + Pdividends) =~ C(K.T)-+K.7) a0 ‘he maturity value ofthis Treasury s K+ CumVelueavidends. For example, suppose the isk-fee rat lS, ‘the stockis 40, and the periods 1 year. The dividends are 05 apiece tthe end of montis and atthe end of 3 ‘months. Then thee present ave. asealcuated above, 89 95¢-*079 = 0.97509 O50 snd their aceumulated value atthe end ofthe ears 0.97539e = 1.02540. Thus fyou buy a ock anda put And sll call both options with strike pices 40, she investment wll be Ke-®™" +0.97599 3.0046 and the ‘aur val wil be 0+ 1.02540 = 4.0254, aang esa aon Sng ea sarge od bug eg my 4, PUPCALL PARITY ‘Quis 1-5 You wish w create a symedc investment using options ona stock The continuously compounded rise intrest at Is 4. The sock prices. You wile 6-mond options with a strike of 4. The stock pays continuous dividends at arate of 1%. The synthetic investment should dupliate 100 shares ofthe stock. Dever the amount ou should invest in Tesi. 1.2.3. Synthetic options an options misprced based on put-cal parity, you may wan to crete anarirge. ‘Suppose the price ofa European cal based on put-cal pars C, but the price is acual selling ats <6. rama diferent perspective this may indicate the put is mispriced, bt et assume the price ofthe putiscorret based on some model) You would then buy the underpriced call opson and sclasynhesied eal pton. Since ats K.0) eke EAS KE) you would sel theright hand sdeof tis equation. Youd sel e~* shares ofthe underying stock, sella European pt option with sake pice & and expiry and bay iskfoe zero-coupon bond witha pice of Ke", or bother worde lend Ke" at he rik-fre rate, These transactions wou lve yu C{S K.#). oud pay "forthe ‘option you bought, snd keep the dflerence 1.2.4 Exchange options Sprion wee stock in eum fr aiferent sce. La, beth va af tne nding se oe fr ‘thie te opin werten and, be the pice the sie ane he oe wich spa Forward vl nw hav a parameter fr thease ()wllmesn a forward gremento purchase asset Q (acl the set wth pce) at une 7 Asuprsit Pl nat» prep oar, before Calls nd pts wl have an ‘otrparametertoe: + €{5;.Q).7#) means call opin writen ate # whieh ets the purchaser let to receive neuen forQr atime T: nother words, to receive max ~Q1)- + P4S;.Qe.T~#) means put option wlten atime f which les the purchaser elect gue in feturn for (sin other words, ree mas(0.Q>~ 5). ‘The puta parity equation then C8047 == PS.QP=6 F(S)~ FZ) an uchange options ae sometimes gven to corporate executives. They are given call opson on the companys ‘stock gansta index Ifthe company's stock performs beter than the index, they got compensated, Notice how the definitions of ells and pts are micor images Acalon one share of Ford wth one share of ‘General Motors as dhe strike assets the sae aa put on one share of General Moors with one share of or as thestnikeaset. In other words AS.QuT=H=C1Q,S.T=0) and we could ve writen the above equation with steals: 8.0. F =~ 810. 5170 Fi(S)~ FQ) 1.2. PUTCALL PARITY 15 ‘xaupie 1D_AFuropcan call opton allows one to purchase 2 shares of Kock with share of Stock at the end ‘of year You are given (© The continuously compounded sisi rate 5% (i) Stock pays dlvidends at continuous ateof 2, (i) Stok pay dividends at continuous ate of 4 (i) The curent price for Stock Als 70, (91 Thecurent pice for Stock Bis 0. ‘A European put option which allows one to sell shares of Sock for shave of Stock costs 11.50. Determine the premium of the European call option mentioned above, which allows one to purchase 2 shares of Stock Bfor {share of tack ‘Awswans The rs-fte rae is relevant. Stock Bis the underyng asst (price $ inthe above notation) and StockAlsthesuke set price Qin the above notation) By equation 1.1) €1S,9.1)=1150+ R(S)~ (0) FES) ~Se-® = 20.6 =57.64057 FE (Q)= Que? = (70.0 = 68.6139. 15.0,1)=115045761737—ea.6199 0 ‘Quiz 1-6 To the situation ofBample 1D, determine the premium ofa Buropen call option which allows oneto Du 1 share of Stock for2 shares of Sock Bat the end ofa yea. 1.28 Currency options {et Clo K-T) bea call option on cumency wlth spot exchange rate! to purchase tat exchange ra Kat time T and Piss KT) the comesponding put option. Putingequation 4 and dela forma in Table 12 together, have the follwing formal Ca KT) Pay ETI 190° Kern? oa) where the “Toreig” riko rate forthe currency which playing the rle fa tack de one which can be Purchased for eal option or the one that canbe sold or «put option) and rhe “domest” risk fre ate ‘which s playing the ole of eash in a stock option the ane which the option ower pays in cllption and the tne which te option owner receives ina put option), Buawnie 1B Youare ve: (@ Thespoterchange rat fo dollars to pounds i 1.48). i) Te continuously compounded risk re rate for dollars. Gi) The coninsousiy compounded sk fre rate for pounds is AS-month Buropean put option allows sling 1a the rate of $1.50. A9-month dolar denominated eal ‘option eth the same sie costs $0223. Determine the premium ofthe$-month dolar denominated pat option, aie pane ange at rape min oth cuenta conta ordre a ten mon 1. purcats pannry -Asswor: The prepaid forward pice for pounds it ne" sins7 ‘he prepaid forward for thestrke asst ols Kee ase 27 = 44479 Tos Claw T)~ Psa KT) 0223 1.4,15,073) 1114,15:075) 002200 1262» [OH] a tbe 44ers (Qala -7 You are givens (0 Thespotexchange ate for yen to dollars is 94/8. (i) The continuously compounded risk-free rate for dollars 5%, i) The continuously compounded risk-froe rate for yen 1%, -A6-month yen-denominate European eall option on dollars asa sike pce of 82¥/8 and costs 1075. Cleat the premium of &:month yen denominated European put option on dollars having ark price orseys “Acall.o purchase pounds with dolarsis equivalent (oa put to sell dolar for pounds. However, dhe units are ‘Bxawnis IF Thespoteachange rte for dollars no euros sS1.05/€. A6-month dolar denominated ell option ‘0 buy one euro a strike price 311/€1 costs $0. Determine the premium ofthe coxresponding eur-denominated pu option sll one dollar fr euros corresponding strike price. ‘Awswen: To sll dollar, the comesponding exchange ate would be $1/€, so the euro-denominated strike prices =0.90916/S, Since were infect haying 0091 ofthe dolar denominated eal option, in dollars ($0.08)0.901) =$0.03658. Dividing by the spot ae the premium neurosis 2°02" gs generalize the example. Let the domestic currency be the one the option is denominated in, dhe one In which the pric is expressed. Let the foreign currency be the underying ese of the option. Given spot ‘ate x exprested as units of dames utreney per foreign cure strke price Kote domestic currency, nd Call premium Ctx XT) af the domestic currency, the corresponding pur option denominated in the fren turreney wl ave sik rice 1/K expressed in the foreign curency, Te exchange rate expressed inthe foreign urrency i 1/a units ofthe foreign cutency per 1 unit ofthe domestic cunency. The call option allows the [piyment of K domestic units for 1 foreign unit. The put option allows the payment of 1K foreign units for T domestic unt 80 Kforelgn-denominated pus produce the same payment as 1 domestc-denaminated cal. et Fy be the price ofthe pat option expressed in the domes earreney. and ithe price ofthe cal option ‘expressed in the domestic currency. Then ra (2h.1)camaan Le Since the put option price shouldbe expressed in the foreign currency, the lft side mst be rnuliplied by xo, resulting in run(Ld.r)-catann Sere EXERCISES FOR LESSON 1 v where isthe rie ofthe put option in he foreign eurency. "Note cat if seemest through cash rather than though actual exchange of cutences, then the pat ‘options KxyP/{1/xq1/K,T) may not have the same paola the eal option Cals K, 7). The put options ‘xP, py at nthe foreign erreney while the ell option Cyl T) pays on the domestic arene. The pao ae emi asd on exchange ate ba exchange atime aye ie ‘Quiz 1-8 The spot rate for yen denominated in pounds sterling is 0.0054/¥, A 3-month pound-denominated pputoption has suk 0.0485/¥ and costs 0.002. ‘Determine the premium in yen for an equivalent 3-monthyen-denominated call option wth ase of veoh, Exercises Put-call party for stock options 11. (CASB-$03s18] A four-month European call option wih strike price of 60s eling for 8 The pice af ‘he underving stocks 6, and the anni continsossly compounded kee ae is 12%, The sock pays mo Aividend. Calculate he value of four-month European put option with sre price af 6, 1.2, Foranondividend paying stock. you ae given: scurent prices {A European call option on the stock with one year to exptation and strike price 25 costs 8.06 (ly) The coninuousy compounded risk-free Interest tes .05. Determine the premium ofa 1-year European pur option onthe stock with strike 26 1.3. A nondvidend paying stocks price 30, You ae given (i Theconsinvousy compounded risk-fre interest rate is 5%. a) A6-month European call option onthe stock costs 3.1. {iy A6-month European put option on thestock wih the same rhe price asthe call onion costs 5.00 Determine these price. 1A. stock pays continuous dividends proportions to its prices ite, Youre ven Thestockprice is 40. G)_ The continuously compounded risk fre interest rite is 4Gi)_A3-month European calloption onthe tock with strike 40 costs 4.10 ()_A3-month European put option an the stock ith strike 40 cos 3. Determine 5 1.5, Forastock paying continuous dividends proportional tts price a rate 5=0.02, you are pve (® Thecontinuously compounded risk-free Interest ate is3%. (4) A6month Europea call option with ste 40 costs 4.10 (4) A6-month European put option with strike a cost 3.20. Determine te curent price of thestock. ese nan es omtnton eet 1.8. Astocks priceis48. Dividends of are payable quarter with the next dividend payable at theendof one moni, You are given: {Thecontinuously compounded risk fee increst ats 6%. G)_A3+emomt Europea pt option with srk 50 costs 72 Determine the premium ofa$-month European call option onthe stock with strike 0. 112, -Adlvidend paying tock has price 0. You ae gven: (© Thecontinuouely compounded rsh e interest ates 6% (4) A6-month Euopean cal option an the stock wih strike 50 costs 230 (ai) A6-month European put option onthe stock wih strike 50 cont 1.30, Determine the present value of dividends pit over he next months on the stock 1A. You are ven dhe following values for year Buropen call and put options at various trike prces: Sake | cal] Tat Price | Premium | Premism cs a wo] so | so_| cis | sar Determine A 200 200 «300 © 320 ey sa 1.9. Consider European options on a stock expiring at ime ¢. Let P(K) be a put opon with sik pee, and C{K)beacall option wth uke pice K. You are given @ RS0)—C15) 635) c¥60 wy 1660) c150 Determine C(60)~ 450 1.10, [€A88-$0026] You ate ven the following: (W Stockprice=$50 i) The isk-ee interest rate isa constantsnnual 8%, compounded continously (i) The pee of a6-month Buopean eal option with an exercise price of 4B 656. (is) Thepseeofaé-month Bucopean put option with a exercise price of 4 is 2. (6) Thestoekpaysno dividends ‘There i an axbitage opportunity involving buying or sling one share of tock and buying or sling puts anda. Calelate the profit fter6 months rom this sete. ‘EXERCISES FOR LESSON 1 19 Synihetle assets LAL, Youre given (© Thepriceofastockis 300. i) Thecontinuousy compounded risk-free interest rte is. (i Thestock pays a dividend a 1.00 eee months fom now (a A3+month European call option onthe stock with strike 4.00 cost 1.20, ‘ow wih create lsstocksyhetialy sing combination of 44-skeoptions expiringin I months and lending. Determine the amount of money you should lend. 142, You ate ve: Thepeeofa stock's 95.00 (0) The continuously compounded ski aes 6%, (i) Thestoek pays quarterly dividends of 0.0, wih the nest dividend payable int month ‘You wish to create this stock synthetically, using I-year European call and put options with tke price K, andiending 9625, Determine the sue price ofthe options 1.18, Yousre given: (i) ‘Thecontinos dvkend rat ofthe indexis 25, (4) The continuously compounded risk fre interest tes (iy) Aaoaday European call option onthe index with strike 21.0 coe 1.90, (0) A9bday European put option on the index with strike 21.00 cost 175, ‘You wish to ceatean equivalent syathetie stock index using combintin of options and lending. Determine the amount of money you should lend. 1k You are given Thestockpriceis 40 a) Thestock pays continuous dvidends proportional tot price st arte of 1% (hi Thecontinuousty compounded risk-free interest ati 3%, (@_At#2day European putopton onthe stock with tke 50 costs 11.00. ‘ou wis o createasymtetie 182-dayTeasy bl with matuy value 10,000, Determine the numberof sharesof the stock you shoud purchse. 1415. You wish o create synthetic 82-day Treasury bil with maturity value 10000, You ate given © Thestack prices. (i) ‘Thestack pays continuous dividends proportional ois price ata rate of 2%, (uA 182-day European pu option onthe stock wth strike price K casts 0.0. (i) 182 tS) Ke. The premlum ofan America cal option must be EilS)—Ke™ + Nogresterthan the price ofthe undeaying ase, since the most you can get wth the option Ite under Ipingasset. Paring these statements opether we get 53 Gal RIVE Coe TIE mFS) KO) ‘The premium of European put option must be: + Atleast, sce doesnot have tobe exercised + least as goats the premium impli by put call pry to cal worth 0. ‘The second statement implies Moy > Ke~T~ FES) The premium ofan American put opton must be + No greater han the strike price, since the most you can get with the option isthe strike price. Nesey setae 2» 2. COMPARING OPTIONS Pring hese statements together, we get ES Pagel KT) PSK T)™ an KE-— RES) ‘Quis2-1 Vou are gives (0) The price of astck 70 (i) Thestek pays continuous vdends athe annual at of 0.08 Gi) Thecontinaously compounded rik-sree interest rates 0.04, Ui) A1-year Amesean put option on the stock haea ss price oF 6, Determine the lowest posslble price for hep pion, 2.2. Early exercise of American options Now we discuss early exorcise of American options. Implicit options rey clloption has an implicit pur opion built aot {et assume tat you ean exeresse acl pion ona stock at me . Fr simpy, lets assume tha you ont exercise the option now, you wil not have another opportunity to exeris the option except a expiry lime T-and als assume thatthe stock dows pay anydvd een mos fan tot he ke DrceisK- Ifyou dont exercise de option nov, then one of two tags wl happen + ther the tock willbe worth more than Kat time T ana you wl ive the stock nl pay K oF + The stock willbe worth ess than Kat hme Pin which case youl hal ono K and not have the stock. An equivalent result can be obtained by buying te stock t ime f for price Ke" (eventhough that not its ‘tue price and buying pu option expiring at time F with tke pice. Then one two things wil happen + ifthe stock s worth more than Kat me T, you wil do nothing and have the tek bt ou val have pak anamountequtalentt thevaleof Kat time T, + fae tock is worthless than you wll eerie the pur option an end up having K bu tthe stock So buying the stock and the pt option is equilent to maintaining thecal option. The pt opon in dhe iterative isthe imp put opdon present in the call option. In reality, you cannot the stock at time for ike"? and this makes the lteative set of transactions have a diferent cost, that does not change the putoption. ‘Simiaiy, every put option has an implicit cal option alt int Call options on nenaividens paying stocks suprising tis that never pay to exercise an American call pon on a nndvidend paying stock ey. At frst your reaction to tls might be “Why not the stocks worth lot more than the strike and youve fal t wil go dove in price, exercise the option immediately and el the stock” Butts ithe situation, you could Justa eal sell the Stock shor and yout ean a es the same profit from the cline nthe stock pic, In fact, suppose you want to exercise the option atime when the stock pice 85; > X. The: "Sing ic sharma ing 6 aru do nt wm Yo ewe ck n't om ya a asd ‘cyan rl Whar sere you ma pee ta nar Wy he pes ube os he Pesongotoalitm ngage ded prs ie cn ot oem npc man ene 2.2 BAILY EXERCISE OF AMERICAN OPTIONS a + Ifyou exes the option, pay K; receive the toc, and sl, your prot is, ~ K regards of how lene ‘thestock pice oes You alo ge interest on ~K forthe period T=. + 1fyou dont execie the option but instead ell the stck short, your profi Sy ~S from the shorted stock, plus mad, 5; ~f rom exercise of te option at maturity You also geinterestonS forthe period ‘T=, The interes this ways greater than trough erly execs since S,>S,—K. IS K the sum of ‘he payos not including ners she sameas the pay wih ary exercise, But ifSp K the payors ‘This tusrates one the disadvantages oferty exercise. Youloso protection against the plea the stock going bel the sre price. The calloptin ges you an impli put option onthe stock Teeter disadvantage of ely exercise hat yon must pay Keates, You lose interest on the se pre, ‘The algebraic proot that eary exercise of an Ampertean call option on a nondvidend paving tok fe not rationals by using putea parity Although pull park only applies to European options an American ‘option is worth at estas mich as @ European option. By pu-cal pay, atime, an option that expies at tine Tsatisies: Coal K T= 1) Pal Ki Pt) FS) Kern but fora nondivdend paying stock, (5) CoulSiKT= = Ral Sin T=1}4S;— Ree = Fan TH) IER (Ie™) en 25k ‘nore than the eerie value, The American option forthe remainder ofthe period worth atleast es much es the Baropean option, so ery exercee snot radonal ‘expression (21), an epresion forthe cll premium, the firs term the value ofthe mpl put the abit to sell the tok for K. The dir tzm, K(1e~"~} ste present vale of interest on the strike price ‘The above does not mean yours hold the option just means that selingthe opon wil et yous higher profitthan exreisingit ear, Call options on dividend paying stocks astock pay dvdends, it maybe rational exercise an option eat. As an extreme example fa stock worth 100s abou to pay liquidating dividend one thatpays ot the entire vale ofthe soek—of 10, andthe strike rloets 60, then the option worth 40 before he dividend, O afterwards ‘We modify dhe above prof that early exercises ot ratonal by replacing the forward pice with the forward price fra stock with dens, F(8)=5, PV (Divs the premiam othe al option decomposed into FaulSoK TOS, ~ K)WPV, (DW) + K (1-8) ‘The put must be worth teat er. Early exerelse wil ot beratlonal if PY, {DW} < K(1— e871) BunPLe2A A stocklsabouto pay a dividend, You are given (0 Thestocks eument prices 110. i) Thestockpays dividends of2 quarter. i) The continuously compounded risk-free rates 05. ‘’ Buropean cal option on the stock expiing in months with strike price 100s worth 11.54 ABaropesn | pu option withthe same condtons is worth 387 ‘Decompose the vale ofthe European cll option into four components exercise value, implicit pu al, salu of interest, valve of vidends Gel SK THE 2. COMPARING OPTIONS Avoweas The ecw §—K=110—100=[10} The European callin canotbe exercised eal but inthis analy th ener aoe x component aa ace “hemp pe worth $7} agen, The alu ofthe pu could have ben dese using put pari. ‘Theale of ere meas te vl of opening payment ofthe cpio 00 ep le rondo) = [5] The nue of vend mesures te vale ofthe dviends the sock pay you exe he opton nom thereradvdend of no and anther dvidend of ote meni, Tht present aoe 2 (10-2). {aoe} dng th componentstogsther. he ahi ofthe Rape pon 104387 165298 11.54, on cling gen va ° ‘BuawP1s28 An American cll opdon has ske pres of. The rsk hee ate, There are? months lett ‘apr. The present value of dividends ove the 2 month period iD. ‘Determine te lowes value of D such tha exarcsing the option eal ould be ator ‘Avowen: Wend D tobe greater than D> k (I-89) 50 (1 e208) = [ATE] a Meaty exercises rational exerise shouldbe right befoea dividend i pad, n onder to maximize the nterest on theta price ‘Quir2.2 An American all option ona stock has asks price ofS and espeslnS months. You are given (0 Thetis eerste isd. (a) Aivdend 1.50 s payable atthe end of today, and another evden of 0 ispayable in months, (ut) Th cuzent price ofthe stock 100. (i) ABuropean put option witha sie price of 5 which expites in 5 months costs #2 {Cou itbe atonal to exercise the option immediacy before the dividends paid? Putoptions[tmayberatonal to exercise an American putoptionon anondviden paying stocky Int the payment of dividends militate against eary exercise, sine the put option ges away de stock ern for ‘ash By exerelsing the option eas you: + Getthecash eal, so you cam nteeston the strike price. + Give up the implicit call option. + Give up dividends ‘The put-call party equation for Burmpean option at timer expiringat time T gives PAS T=) = 8, KTH} + Re" “5, + PDI) C15, K,T— ARS.) +PMDis)~ Ke") ‘The second term isthe exercise vale. the other tems ed up toa negative, Iemay be tinal o exer eat (ou cannot be suze, since the putea party fora Earopean pt an an Amedcan pt may be worth more) "Tosummarize Torstocks without dividends + An American call option is worth the same asa European call ption. +n American put option maybe orth more than a European put option. ee 28. IME TO ENPIRY a 2.3. Time to expiry ‘An American option with ime T to expiry must be wort at ast ss much as one with time < Po expiry since you ways have de option of exercsing the onge ved option a time t tis not atonal 0 exercise an American call option on a nondvidend: paying stock ary, an therefore European option equal such an option in value. Therefore fortwo Eurapeanclptons ona nondvidend- paying tack the ane with the longer Hime to expiry mst be wrth a last as much as he other ne However fr European calloptons on dlvidend-yaying stocks regardless of whether dividends re disrete ‘orcontinuous~and forall ropean put options, no genera etatementcan be made + For European put option longer time to expiry may make the present valu ofthe cash received lower + Fora European cll option on a dvidend-payng stock, here may be a lange dividend jn he future In ‘that cast shorter duration option, one that expires before the large dividend, would Ukly be worth ‘more than long-duration option one that expies after the lange dvdend, tn the mos extreme cas if ligudating dividend one tat sequal to the vale ofthe tock paid at time ¢ +e, Europe eal ‘option expiringat time Is worth more than one expringat ime T> he strike price increases tthe ssf ate, ten the premium ofa European option on wondvdend paying stock cannot decrease as ime to expiry increases. Tle rue forth eal and put options. Ntie that forcallopions on nondividend paying stocks, his ttement stronger than the one inthe previous paragraph, ‘Compare thew statements Previous paragraph ‘Thieparagreph For two Furopean call options on For two European call options on nondivdend-psying stock, nondividend-psying stocks withthe wit de longer duration ane having strike price equa othe ‘ame strike price, the one with the Shower duration one accumlted at intrest forthe diference longer duration to expiry must have of durations, the one withthe longer duration to expiy must, value atlestashighasthe aterone. haves value atleast as high asthe other one Foracalloption, thehigher the tik rice, the less able the option, so de statement the sight is stronge. Since the stock does pay dividends the same statement sue for American cal options but not acess forAmeriean put options alonger-ved American cll option with site pice increasing athe ine ate ‘must he worh at eats much asa shore ved option. Before working ou the deals ofthe proo, her the inciion. Lett < 7, and consider calls C{S,Ky)and {S,er-9,7) HC(5,Kyt) is ore expensive, sell rand buy Cs, Ke"). Ate f, the worst tat ean hhappen that you may recave K and pay, but by time T,X wil have own co Ke", and at the very last. youl ables return Ke"? and get back. Sou can possibly lose money by seling C1, K) end buying (8.69.7) and you may actualy make money. or pts the samelogic applies. Consider puts PUS.K.£}and P(S,Ke'™,7) 1S K, amore expensive, selltand buy PS, e"~,7), Ate the wort thatcan happens thatyou may receive Sand pay K. Atte 7, youean eur Sand receive your K back with Interest, 50 you can possibly lose money and you hay make money. Now le’ go through the deal Suppose for nondvidend paying stock Sell opdon C{, Ke", 7), wth T >t, costes than CS, ‘Then CS,K, )istoo expensive so selitend buy C{S ke", 7). You immediately gta cash fw Of C18 Kt) {S. Kor, 7) Now ers evaluate your future profit under a possible scenarios 1, Attime tS, . Then C{S,K.#} is wonhes. You keep the profi yu already made. In addon, you may ‘make additional prot 5; > Ke'™, | uy C18. 8.7) Sol 6S, i?) Net reel af C5.) C48. Kr.) "Time Pay :—Ki Sallstockshort. ecreceip 1S. Kit) worthless Time Buy stock forSr. Netpayment; K.orow the stockand deliver an callect the se price K. Alternatively, payoff; and el the stock shor achieving the sre est, since you gS, forthe stock and pay 5, ~K, leaving Yyourwith K-Inves Kathe isk fee rates youll have Ke"? at me 7 Tens (a) If Sy < Ke’, your ecumulated cash wil be t least enough to buy back the stock and you get ‘adaional profit tothe extent the sock pric slower than Ke", (&) 1S; > Ke", by exrising your option o buy the stock for Ke", you have ust enough cash tocloseall transactions at no gn or oss, schematic ofthe above transttonss shown a Figure 2. ‘Thesame oie works for puts. Suppose for nonlvidend paying stock putoption PS, Ke", F)costless than PIS Kt). Then sell 1S, K,¢} and buy MS, eX?) fran mediate gai ofthe difleence in pices. Then 1. tS, 2 K, PS, 4.1) expres worthless and you potently ould make addional profit fr < Ke", 2. I, K, take delivery of the stock, o sematvely pay XS, and buy the stock ther way you spend K: the second way, you spend; for the stock and KS, pay offthe option, Banwow K.Atume T You owe Somaceane 23. TIMETO BOY oe et?-9, Then (@) WS; 2 Ker, sell the stock and use the proceeds to pay off the lan, You pt ational profit of Sreke9, (0) fs, <.e", the payoff of yourpot option is ust enough to repay the loan. ‘xawpi#20 Fora nontvidend paying stock with pce 21: (0 The continuously compounded rskfee rats 4% {iA uopean3-month put option on the stock with srk price 20 costs 1.0 al European 6-month putoption on the stock with strike pice 20:30 eo 0.90, ‘You ake advantage of arbitrage. Assume that you sll one 3-month put option and follow te optimal sta ‘ay and that he stock prices 19 afer months and21 afer months ‘Determine you et prot ater months ‘Avs By the loge we ust went dough, a Buropean6-month option wlth sake pice 20690) = 20 2010 ‘sworth moze than a European 3-menth option wih srk pice 2. A higher srk price makes 2 put option ‘wort ever moreso the 6-month options certainly worth more than 00 ‘ou sell one 3-month pu option and buy one month put option or an immediate gain om 0.10. Aer 3 ‘months you pay of forthe option and buy the tock fr 19, otal cost 20, whlch you Boron. After 6 months yousll the stock for21 and your option s worthless The cash Hows look ike this Sale/payorf Purchase of Purchas/salo Ne Time _of3-momhpur_G-month put ofstock cashflow montis 1.00 030 = 010 Smonths 100 = 1900-2000, montis 21002100 ‘ets add up the 3 cash ows accumulated at interest 0.1020 202010421 0000: a ‘Quiz23 Suppose evo call or put options CS. 4,8) and GIS, Ke, T) ona nondvidend paying soc have tho same price You ean then sel (5K) and buy C1S, Ke", 7 with either gin nor fos ad with hs poston, you would then be guaranteed no to lose mone. we ignore the posses ofS; = K and 5; Ke", thee ae fou eases: wo possible fos atime tmes tw possiblities or Sat time T. For sone ‘ofthese possibilities you wll make money. Flin the following boues wih checkmark fo the cee where you make money. cals Pats Sick 52K Sick 155K sreKet™n serene 10686 an 994024 21, sro keno Spo Keto ‘quia-a A nondvidend paying socks caren pies ¥. You are givens (@) The continvousy compounded sks incerest ate 5%, {a} A European 6-month call option on the stock with srk price 45.0 costs 0.50 {ty A European 12:month ell option onthe stock witht price 46:14 costs 0.55, (You crete poston which takes advantage ofthe arbiage betwen these two options by buying oF seling exactly one eal option ofeach pe 2. COMPARING OPTIONS ‘Table 2.1: Summary of relatonthiss of option prices and time to expiry. In these rules, > |. An American option with expr T and sie price K must ost atleast as much as one wih expey¢ and srike price K. 2. A European call option on a nondlvidend paying stock with expiry Pand trike price K must cost. atleast as much as one with expiry rand trike pie K 3. Aburopean option ona nondividend paying stock with expiry and strike pice Ke" mustcost, east as much ase with exp fad tke pice K ccallpremium 225) 0 15| 10] as| So eo wo 30 35 CT ca a Site Price Figure 2.2: Call premium as 2 function of strike pie. Assumes $= 50. Calculate your profit atthe end of 12 months ifthe stock rie Is $0.00 after § months and 47.00 ater 2 months “The rules of hi seoon are summarized in Table2 1 2.4 Different strike prices 2.4.1 Three inequalities ‘The hre inequales we iscussin this section apply to both European and American options "To eta eel for how option prices vary with sek pices, look at Figutes 22 and 2.35 sypial examples of ‘option prices for various strike prices? ‘We wl discuss tree inequalities fo premiums of options with diferent sue pees which apply regardless ‘ofthe option pricing model. While option prices may not be a dieentable function of the srk price in an ‘citrary pricing mode! we wl nation to stating hese inequalities algebraically, use derivatives to desde ‘these Inoqualies since this may make them eal to remember. The fst two inequalities involve the st ‘erivative snd the tind one involves the second derivative. tS ety lets mine nga 23 pe sn ae 24, DIFFERENT STRIKE PRICES x Put premium ——— 7 3s 0 6 30 35 6 suk Plce Figure 2.3: Pt premium 35a function of sve price. Assumes $=50 ‘We wil also strate arbltregs tha can be crested if thse relationships dont hold. tan abitrage exists, ‘hore are many ways to eeatet Two aye ate 1. Greate position which results in maximal immediate gn, and which cannot possibly loce as much as ‘he nia ain inthe fare, 2, Createa poston which sls in minimal immediate gn, but which asthe possibility of ature gain, ‘We il itustrate bot ofthese ways Direction "The ist thing you notice in Figures 22 and 2 ls tha the graph is monotonically non increasing i the call graph and monotonically non-decreasing in dhe put graph. Ths ls always ue. Fora ell option. the higher the strike pric. the ler the premium. For a put option, the higher the rks price, the higher he pm, Algebraicaly for Ke > GS. SO18. Kis) AS Key T2P(S.K.T) ‘wth derivatives: 208.6. « Innulvely this is obvious, Fora eal option, you must pay the. Pricemeans you must pay nore, decreasing the value ofthe option, Foraputoption,youreceve these price ‘upon sling the tock. The more you recive the higher he value the option ‘ats reat arbitrage fo alls. Suppose that fr K: > Kj, C{S. KT) > O(5.K, T). Then O(S Ke Ts ver ‘priced, Inany arbitrage cll the overpriced option and buy tie underpriced one. 2» 2. COMMRING oPriONS ‘To create the st ype of arbitrage, sel one K; option and buy one; option. You have an immediate gain (0 (5k, 1)~ O15 hy, T) and no posto futur lose. Let # be the ime of exercise ofthe Ke-stckeoption; you exercise the ky stke option at tie I's, = ;, both options are worthless I Ky <5, =f, he option you Sold is warthoss andl you get a payol on the option you Bought. IS, > Ks, both options pay of but the option ‘you bought pays Ke ~ more than the option you sol ‘To create the second type o arbitrage, sel one Ks call and buy CUS Ki, TCS, 45,7) Ki-strke cal ‘Then costs0, ats = The theme of werelse af the Ky-suke option. Exercise he K;-se option at ine + aswel IS, ©, none of the options pay of. K, K; all he options pay offbut she options vou baught pay mare. {quiz2-5 Two 6 month Faropean put options on «sock ae avaliable withthe following tke prices and pre ‘ike Pice Premium 4s 4 You are als given (© The eurrent stock pric is 47 (i) The continuously compounded iskfee rate is 5, You take advantage of abitrage by selling a 35-srike put and buylng45-sike puts so that the net cost is 0. ‘hermits the stock pie 32. Calculate your total prot ater months ineluding imerest siope ‘The nest thingyou notice in Figures 2.2 and 2. thatthe absolute val ofthe slopes es tha 1. The premium {foracallopion decreases moreslolythan hear ree nee. The prema fora pt option increase ore Slowly than the serie pic inereases. Algebra fr K, > K, 1S Ke FI-C18.K 192 KK AAS, Ka TIPS. KisT) SK Ki ith derivatives 20.5.1) ax? 20.4.0) oe Inu hiss clear. the strike price ofa call option is decreased by a the best eases that the new call option pays a more than the old one at expiy. That ste most adional amount ean pay. and may not ‘ten pay that much ations for example its wortles a expiry oF wort es than a at exp The sane Togle works for pur options: increasing the sike price bya a most causes the option to eens ts payout by Let create anaritage focal, Suppose Kr > Kj and CIS. Ke, 1) Ke, youpay,~K; and ecelwe 5, ~ Ke for ant costo K,~ Ki, whlch ess than you ina a, The second type a arbitrage i lstated in he Fallon example 24, DIFFERENT STRIKE PRICES 2 ‘Bewwrur2D Two I-yeerBuropean call options onthe same stock re priced as follows ‘Suike pice Premium o 10 ‘The continuously compounded rsk-tee ate 008 ‘ou tke advantage ofsebitageby buyingone 45st cll and sling 40-eke cal, where fhe lowest possible valu that results inno poss los when interest i nore. “Aer one yeas the stock price 6 Determine your profit including interes. Avswen: Your inital gins 10¢—, We sleay saw above how to perform an arbitrage for = Therefore, the tomes value fe that will wrk cannot be more than 1. Now consider the payolTat the end of one year. Well consider the thee cases +s) 45. Thecall ou bought is worthless and you have to pay eon the calls you sold, 4+ 5,245.15, =45 4, > 0, then you collet kon the cally bought and pay e+) on the cls you sold. The nec payments Se + ke 1} but tis sess than Se sine < 1s this ease oss yo less han the fsteate + 8,45 fS, 40 then all options are worthless and there fno payoff, ay Ee, hich less than Se, 2 this cae cots yo les than he fst case 404k,0 Ky, andthe contin ‘ously compounded risk-free interest ats. Determine the maximum possible value for C(,K,1)~ 18, KT) 2 Redo example 2D, except tet instead of seletng cas the lowest posible value that resus inno poss bili oss when interest ignored selec asthe lowest possible ale ha ets inno possibly of oss, taking meres into account —— 2. COMPARING OPTIONS convexity “Looking atthe graphs in Figures 22 and 2, wesce hat they rw bow-shaped, or concave wp. The put premium Increases more gull wih higher K ad the call premium decreases more slowly wih higher K, Option pre- ‘mins ar convex. The ras of decreas in cal premiums asa function of K crease. Th to increase at ‘premiums asa funtion of Ktncteases Ofte tre properties, dis ne isthe hardest understand Soles stat witha specific example Although the following example is phrased asa European option, everthing apples equally well o Americ options, shee fone option s exereied early, che ober 2 option could be execsed atthe sme time ‘Suppose there ate 40th, 50-strike, and Orie call options on the same sock with the same expiry date T. Let portolioAhave one 80st eal option. Portolo B wl have a dDstrke cll options and 0, Ste eal options. We want to make portfolio pay a least ax much ax portfolio A inal cases, What i the ‘minimal (a,b) such that portfolio B wil aways pay at east as much as portfollo A? By mini mean his sit ‘he ordinary definition of minimal, butt reatonable in this contest there sno pat (e} wth ea orth (= and 00 1. 8(7)< 40 ial since none ofthe options pays, 0 BS payout equals AS payout in thet ease no mater what (a8) 2. 40 2, i he payots onthe wo portfolios increase at the same rte a SIT) fetes, ‘hen portfolio B wll aways pay atleastas much sportfolo, since they lead pay the sameat 0, bythe las paragraph. Conslder the change in pay as (7) inereases bye. For portoio A. the payor nerenses bye For portfolio B the pay inceass by efa +). Sowe need a+b = 1. This means B=025, Pure 2.4 shows the resting Por B when (a,b) =(0.7,025) We see loking atthe igure that b was set so that the ine ging fom (0.0) hits the Portola Aline st SCT) BD at which point the O-stike option ays lf. With (2.0) =(0.7325), porto B pays at leas ax much as portfolio Ain all cases, but pays tore ‘wien 10 < S17) 0, Portia pays nothing below 0, nd pay less than pordalio aoe 0 but brow 80, So Pottiolio B must costatleastas muchas portfolio 8 That means thatO75C{S 40, 7)+028C(9,80,7)= C1850.) The generalization of this loge that > Ke> Ky and we express Kz = aK DK; wth ab =I, then (8.4.1) = 015 Ky, T}-+BC\S. KT) In oder words, the raph fell prices a function a Kis below Straightline, or conver yu draw stright ine rom one pint ofthe raph to another, the graph wl alays bbebelw thane, And hee tre or ute aswel, ‘Algebraealy for Ky > Ke > Ky CUS.K N~C1S. KT). C1S.K T= C.K.) co Teak. PAS Kas PAS.KG.T) USK TI PS.A.T) aR Tek [Notie thatthe factions inthe frst equation are negate 24, DIFFERENT STRIKE PRICES a Paso ry 20 ol ‘0 * 30 o 0 wo Figure 2.4: Demonstration that Portfolio B, consisting of 0.75 ofa 40-strike call option and 0.25 of an {0th call option, alnays pays more than Porvoo A, which consists of one Sse call option ‘An equnalent way of expressing commen iat the price of the Kt option estan thee Incrpolted prie ote ober oon: BK SDS. 72 1R = KOA 8.0) n Ps, s% Interns of derivatives, {Quiz27 Tor options wit the same sie (European or American) and epi dave, you are ves (strike put options ona stock have price 6 70-ske puroptions on astaek have pice 10. Determine, based on convexity of option pies, the highest possible price fr 0-stike put option and {etermine the number of 0- and 70-stke pl optons needed to guarante a payof at expiry et east ae age asthe payolfofone 6-strike pt option. there sa mispricing, the option inthe mide wil be overpriced elatve othe other wo options. There fore, sel the option inthe midland buy the ones athe two extremes. ‘Theis sype of arbitrage isthe one where you maximize nil gan, and that bth one we have fst die cased, To reat the st ype of arirage with call sel the Ke-ste cal. Buy spk and Ks cls such ha inst combination ofK and equals, Buy B=E ky stalks callsand 2 K;-stikecalls, Since ‘hereisa mispricing, the price ofthe Ks eal igreater an the nearly interpolatd price ofthe oer wo 2 2. COMPARING opriONS cals co ou have an Immediate gan. Suppose the Ks opon i exercbed a tine. Yuen the other options ihe same tine If Ke Ke opton swore so you don pay ating. TK < 5, ~ KYU poySy= Ks andvecve BRS) Ki). Note that or 5, = Kyte mount ou ay the aout yu eee. ‘The amoun you pay decteasesby for every dap of nS, butte amount you cee drops by Be <1. 30 youavea net al If < Ky, Forever 1, ceases vr, yu pay an.addlonl 1 and ecehe Sf Hom the Ks option and om the X-sike opto, wich adds up 1, 5015, > K;Yourne est 0 at “The second rp of blnage isthe one where ou pay nothing now but guanine. Trent he secondtype oats ell oe Ky sie ca andy the mulpe ofthe Kak and Kes calls eed Intent peto maketbe inl gad, Tis muliplevalbec> 30th parol you Bought wl pay teat Crimes as micas the one you so -Exoerte2E Fortes 6 mont American calopionsonastck 1) One wih tke price 4 sels for 0. (0) One it se price sel for700 Gi) One witht price 10 sel for 250 “Te option with sie price 4 overpriced Iased onthe convey Propet of option premiums. You therefore slit Determine thermal an the minimal amount the the? options you sould buy to arnt a prot Answese Letbe the amber of40-sikecallsand y the number of45-stke calls. To assure no ntinvestment, ‘weneed 800 +6.30y =7.00 assure no loss, we will eerie our options when the 44 ssc one exercised. the stock price Sis ess than 4th 4e-snke option wll ot be exerised. It isbetween (and 43, we need a(S an) s—a8 4 20 and the worst posible case = 45,50 J, ube stock prices greater than 45, wo eed AIS —40)+ (8-15) 28—44 Sixty —1)—s0r—a5y 45420 Tor $=45,y drops out and the inequality essed for x= J. f+ y—1 <0 then the left hand side isa decreasing linea function ofS, s for suficentl high the inequality i vosted, Conversely, K+ ~1 20, the left hand sid a non-dereaing linear function of, softs grater than Oat S= 46, ti cetany greater than OforS> 4, Sowe conch that he inequaly esas forall $> 4S and only sy = 2, 9508-46: <7 3 xty2h ‘Aves the highest possibe value fy som the ist condition) (7~9.50(2)/6.30 0.0552. The Imersee- ‘on a the stand third conditions i x=7/52, y=25/32. Therefore, x can be between 1/5 and 7/2, while y ‘ean beberween 25/32 and 1.20852, “The values of x. allowing arbitrage ate graphed in Figure 26, a 24, DIFFERENT STRIKE PRICES. aE a ws rr) Figure 2.5: Vales of (x,y allowing atbitage in Example 26 2.4.2 Options in the money ‘An option inde monet would have a postive payout oul be exercised Acallopon inthe money {tthe rc les tha he nding ett pe. Apu option sin he money teste re more ‘han he underyingesset price “an opton sou of the money the prc ofthe undryng eee deren tom the sik rc In och 4 way tht the option doer py of eosin ou othe monty tes poss ree tan te Uisedying asset pce 4 put option owt of te money the sep lt tan th undrng ase Pe “Given wo Amen cllepions wth he same expt te money CSR? an C1. 7) with > Mees CUS es optinal ten sols exrchng CS KT) anoptioneven moein the ane, Th une ts for pus: tS 7) and MS.) arent money wth > Hy an exring SK 7) opt then i exercng AS kT) an option een moe nthe money, These atements ll fom the second Proper above or eample, rene 1. The profit kom exercising CS, Ke, T)ieS~Ki. 2. By he second property above, OS, TS K-08. 4.7) Kao K 45K =s-K, ‘which isthe exercise value. Sot best to exercise the option. a Ne -mcon 2 COMPARING OPTIONS Exercises Bounds for Option Prices 21, ‘Thecuren price ofastockis35. Let USK, 1) and PS, K,7) be Buropean calls and puts respectively on the stock with eke price K and expiry 7 ‘Which othe flowing statements are tut 1 MS.5,7]235e-1F—a5e-6F IL AS,35,7)~(8.90,7) 2806-7 354-8 ML P35, 7)~C18,90,7)235¢-"~38e"8T 22, Youare gen: @ Thepreeofa swockis3s {uy The staek pays continuous dividends proportional ots price atan annual at of 0.0 (i) The continuously compounded rsktte interes rates 0.06 (i) Aone-yearAmereancalloption onthe stockhas a ske price of 2 _Deteanine the lowest possible price foe his cal option, 23, Youare given: W Mepaceorastockis (i) Thestoek pays continuous dividends atthe anual ate of D4 (i) Thecontinuousy compounded risk ee interes rte 0.06 (i Aone-year American put option onthe stock has a stke pice of, Detemine the lowest possible price fo this put pson. 2. A 12-day American cal option on stock has strike price of 10, The continously compound lk free inerestate et ‘Adhvidend is payable onthe stock on day. Determine the lowest dividend for which early exercise of the option may be rational Diterent strike prices 25, ‘ou are gven telling pres for American all options: ‘Wake pice | Option pice 0 2 Determine he highest possible pie foran Ameria callin with strike price 8 pit een ercontnon hen 26, Youaregven the following prices for European pat options: ‘Sires [Option price cy 7 a 8 Determine the highest posible pice fora Buropean put option with strike price 4 22. You ae iven the following prices for American cll optons ‘Sikeprioe [Option price @ 5 50 4 ‘To capo the mspricing you sell one 0-stike option and buy = 40 seke options andy strike options, wih andy selected create an arbitrage. Determine the range of possible values for x. 28, ‘You re given the following pres for Amerlean put options: (Sik pie | Option pee 50 5 ( & 2 = | 6 “Teexple the mspricing, ou wll uy ten 50-srke put options Let be the number of strike put options to sll andy the number of75-stke put options to buy. Determine the range of posible values fox 29, Forastoc, you are given: ) Thestock’s prices 0, (i) Thestoek continuous dividend rat f 0.02. (i) Aone-year 5-sike European cll option has premiam 10, (i) Aone-yea 45-suhe European call option has premium 2 (0) Thecontinously compounded iske interest ates 005 Determine the lowest and highest titrage fee premiums fora I-yea4srke European put option onthe stock. 2. COMPARING OPTIONS 240. Forastock youare piven @ Thestoek’spriceis 30. (6) The stock pays no dividends. (i) AB-month 30strke European pu option has premium 3 (A month 0-ssike European put option has premium 6 (©) AGmonth 45-srke European cal opson has premium 3.5. (si) The eaninuowsly compounded sere interest rt is 0.05 ‘To exploit the mispricing, you sel thee 4srke put options, buy one 20-srke pt option, and bay two syhete 4s pu options created using appropriate amounts of 4.snk all option, share of tock, and kfc bonds Calelate your ne gai fer 6 months, neuding ners. fhe stock pees 4 then, Additional released exam questions Sample2.25,CAS)S7-12, CSS-10713, M/S S012 Solutions a1. 1. By puteal party PS.85,7)~C18.38,7) ae 1. By puteal parity AS.30, 7) O(8,10,7) =e"? 356-7 and AS,98,7)> AS 30,1). ML, Since BS 35.7)~ C(8,38,7) = 85e-1¥ ~350-* and C{S.30.7) > 6(,38.7). the inequality should be re- versed 5.35, 1) O15.90,7) 535-7 356-87. X 122. Sine It san American option, could be exercised immed for agin of 35—32 3, 0 it must be ‘worth atleast, but weean dobeter itisworthat eastas mich asa topes ealloption which muse worth atleast as mach asthe pu-eall pay value using put value of, or — Ker =ase 206 28, An American put option i worth more thanx European put option. By part to «European call option ‘wah value the put option ssworeh a east 135 34,10 Ke"? Se We"? 35°F, and C(S88.7) 20, $0 NS,35,7) 2 356" — 195.92,1)2 et a562 6076 “Thatowerbu inet ax onda the eran] ne oy ower oun forthe pt eo. 2, the very let ety exer ensigns on these price Paying dy 1 cont 10 whe shops yo ingyen he nee sto (te-®) = [36807] sothe ivdend must beat least 0995017 25, Byconvedty, 15.49) <02648.40) +0015.) o202)+-000)=[3] 26 By convey, (Settrsine ($212) ts.0)= 000 v0.4 8 eee EXERCISE SOLUTIONS FOR LESSON 2 a 2:2, Reforto Example 2 for asim question. The constrains her are setyst ay! ‘Thelnterscton of thes! an third constraints i (1) 2 28, To have no los inital, we need she proceds tom the 6-rrike puts (12 foreach put sold to exceed the cost ofthe S0-trke puts for each ofthe ten purchased plus the cost ofthe 75srke pts (fr each pt purchased, or 2x2 10(5)+ 15 2-187 250 he 65-srik pus are exerlsod, we can exercise the 75-srke pats smultaneousy. To have no los i the 65. strike pus are exertsed when the stock price between 0 ands SD beng the worst case) erect ainon 1 Tsk puts to exceed the payment on x 65stuke pus since the 0-srthe pts may end up beng worthless) 65-50) 5 15-0) xsiy ‘ohare no sth sie pus are xed when the ock ne ees than he wort pos cases itsesockpetn We sasumethe sock pce canotDenepe) Then tenon ens0srke pus any "ene pas moat oxedthe penx 6 ae puso son0)+ 7572650 e757 <500 tae 15y=100 ‘The intersection ofthe fst and second constants is 12x—15y=50 sal 12(fy) -157=50 sy=50 ‘The intersection ofthe st and hid constraint ee 15y=50 ax 15y-=100 0 12660) 50 _ 10 1s 3 ent onan 8 2 COMPARING OPTIONS 7 w w a w Figure 2.6: Values of x.y) demonstrating atbitrage in exercice 28 ‘Te intersection ofthe second and third constrains (not needed to answer the exercise question is igure 2.6 shows the area of possible}. Thus 0/38 80 28, By slope. the difference berween 35k call option and a 4-srke call option fora European ation ‘annot be greater than 5 eiseounted at interest See Quiz 2. Therefore the largest difrene of pice between thocalis5e-""=5e-% = 475515. Thisimpllesthaas0-skeeal option must be worth teas 10-4 75615 By convey, a 40-strike all option must be worth t mos the linaty interpolated valu of 35-strke and «5. or ()(10) + ()(2}= 6. By putea par. the put premium s 140,401) C140,40.1) +406" — ave s0,40,1)-—1.45077 Pliggingin 524285 < C14, 40,1) 6, we ge KOS" (G04 1) SaaS 210, ‘Tosyntheszea45-suke put opton,bypuv-eall pari, we need P{40,45,05)=C10,45,05} 4456-5 so:o synthesize 2 puts, we buy 2 cals, make loan for 2(45e-MS = 47.779, and sell? shares stock. The total cash low of the nil ansactons ie (QUIZ SOLUTIONS FOR LESSON 2 0 Buying20-strke put 3.0000, Seling340-srke puts 180000 ‘Buying245-sukeealls —7.0000 Makinglosn a Seling2 shares oac00 Toe oz _Afer6 months alloptions are worthless. Theloan paysback 0 and we pay 2(¢2)= to buy back the tock ‘heii gain saceurulated with intrest. Thus the ne gain ler months ie ‘Quiz Solutions 2:1. The American put option must be worth eas & much asa Buropean pt option. By pial party ith call premfum of: Pug Ker? S04 2696-97968 1.6763 So the American put option must havea price feast [1.6763 2-2, Thevalue ofthe putoption i 0.2. The aki ofthe interest onthe stke prices a5(1-e 2) = 1009 Sothe total loss by exreisingnow is 082+ 14049 = 22249 "The pesent value of dividends is 15 (12-005) — 29851 Since the present value of dividends seater than the loses fom ery exerts, Irma be ratonalto exercise the option cai 2-3. Youclearly make money the boxes where thet option wortles andthe T option pays ot These ae the lower et eal box andthe upper right pot bat. In adaon, the boxes where the option pay aff and he 7 ‘option s worthless ae, paradoxseal boxes where you make money. For acl, ifthe Faption pays off you sel the tock short and receive K, Ate 71S; < Ke", you pay less than Ke" oy back te stock and keep the excess interest, Ke'?~~ 5. Fora pat fhe f option pays of you pay K and teceve the stock. At time 7,115, > Ke"), you sellthe tock and ave more than enough to pay back the loan of K with interes ‘othe folowing are the boxes with checkmark alle Puts Sick SK Sick “So Sr Kero 2-4. You sell 6-month cll and buy 2 12-mont el for gain of 0.05, Aer 6 mont, thecal you sold is exercised. You receive 4500 and borrow the stock odelver I, or alternatively you sete for 800 and el the ‘tock shor for 8000, ora net cashflow of 4.0. 6 months ater your 45.00 Is worth 4Se=" — 46.14, You ‘exercise the 12-month ell pay 46.14, getthestockbeck, and return the lender, or lteratively you sete by ‘receiving 0.86 and buy the stock back for 47.0 fora net cash low of 4614, Thefllwlng table shows the ash flows: ‘Sle/payeffof Purchase! Salejparhase Net Time _Gmontheall_12montheall_—ofstock cashflow ‘montis 06 “058 — 05 Smonths 5.00 = 000 48.00 Rimonths 06 0046.4 2. COMPING OPTIONS ‘our prof she 0.05 you gained athe beginning accursed with a oar nterer 56" 45.0069 96,14 [OBE "Note that your prot would be greater ifthe fina stock price atthe end of 12 moms were less than 46.14, since youve sold the stock shore after 6 months. You would bu it back a at thei stock pric and the difer fencelerween that and 40,14 would be additional pro. 2-8, You dont ned he interest rate othe nt ck pce. ‘ou buy 125 45seke options so thatthe net cost i 125(4)~S= 0, Aer 6 months, he 45-ske options py 125(5~82)=1625 andthe 3.strke option pays 35~32=3, Ne profits 1625 26, 1. [E=77OE =I] the maxim valve. Otherwise one could buy the Ke stske and sll he Ky she, At xp, he Tia gan wl have grown at interest oan amount greater than the difeencein strike prices ‘2. The inal ain accumulated with interest is(102~4)e0™. Therefore we need (ve =a) ~Sem0 eet 5) ae 40m Toe ‘Te inal gan 100 742888) — 4-3-4288, Ateapey, the gun i (46 45)~0.742084(46~40)=~24579, The ot gan at expiry 3.428046" 3.4573 127 We ned to equ the payofs a 40, the wort cane the 40-70 range, 20 we need 2/9 a 70-the ‘option. Then we need 1/3 of 40ssre opdon to equate the pays Blow 40. "The price ofthis porlio is 4(10)+ 0)}=[Z]} so1he oosteike pu option cannot have a price pester th 7 Lesson 3 Binomial Trees—Stock, One Period Reading Derisatoes Markets 10.1 | expect many exam questions fom the material on binomial Hees, particulary the material inthe next wo lesrone During this couse, we shall earn to ways to pice options. The fst way (binomial resi discrete and ‘oclly general but computationally intensive. The second wy Blick Scholes) analy, bu requlzes several assumptions and cannot be used general, Ths lesson ean ntoditon to binomial ees. ‘Ina binomial tee for an option, we beak the tine to expiry ofthe option int periods In each pti, gen the price ofthe underying asset at the beginning ofthe perio, it en oly move to one of vo prices atthe end ofthe pesiod. Wecan then determine the value of te option recursively by startinga he expiy date eluting, the value of te opdon under each possi for final price of te undeiing asset. and then moving backed through the te. “To star of simpl, we wil assume the underngasset isa nondvidend paying tock Thebinomal ee wll have only one period. In other werds the stockeun oaly have one of values at expiry ofthe option, ‘Conder the folowing example: Fora yea? European ell pion ona nondivdend paying ine Uh) Thestoek prices current, (t) The stocks price wil be ether or 0 atthe end ofthe ye. (i) The eke price 55. (The continuously compounded risk-free rat is “Thebinomial tee forthe stocks a ~10 3.1. Risk-neutral pricing “Temporary imagine a risk-neunal wodld. This means, for example tha given a cholce between having $0 for sure and having a random amount of money wih expected value 0, one i indiferent. One docs not expect 8 risk premium—an expected value higher than 50—to makeup forthe lack of certainty ofthe pay, Ie so ‘means that nei sts to ear the sf rat on a sky asset. Tina rsk-noutl word, we can mediately calealte the probability that the stock price in the above tee wil go up 160. Namely the stating price othe stock 50 must equal he average ofthe present values of and ‘so eight by probably. Lat p"betherisk-neutal probability hat the tock goupto60 The ermembes, Teen too a en clinica hah ne pein aspen ‘the riko a 558% andthe periods one yar, s the dscount actors 506 (009 +4001~p) e209" +0) suet (528186 the Hsk-newcal probably that the stock wl go up in pric ‘Lets now sess the option we want topic. The 1-year call option has strike price ofS. This means that {he option wl pay ithe stock price i 60 and ofthe stock prices sat the endof the year. Wecan update out twee by placing the option values below the sock pices ‘Asin he previous lessons, {35s the call option premium for @-year option witha sike rice of 55 ‘Using he risk-neutal probabilies we developed above, the option premium must be the expected present ‘valu ofthe two ending pices, or 155,11 =6°*((046281845)+(1-0.62818)0)) =29877 ‘So the option premium is 2.877, and she wee looks ike his ‘quiz3-1 1 rear Buropean pu opion on a nondidend paying stock as strike pric of 0, You are ven ( ‘Thecurent tock rie s50, i) Atte end ofa year. the stock price willbe ether 40060 Gi) Theris-reerateis 5%. Desernine the premium forthe put 32. REPLICATING PORTFOLIO sa Figure 3.1: 3 period binomial te with fa call option pays off at both the upper and lower nod, then the pu option withthe same expiry ¢ and strike price i worthies. nis eas, the Value ofthe cal option, by put ell pay toa pt of value UK. 1)=5e°8— Ke" Similar fa put option pays off at bth nodes, ts ah ks PK. )= Keser ‘We now introduce the nation w and d Inthe binomial re ta indicate the constant to multiply the ntl ‘value by to get the upper (u) and lower) nodes. nthe above example, u 1.2 and d04 so that thei bree $= 50 goes ether to uS= (1.2150) = 60 or dS = (0830) When we generlze to amul-prid tree, tt and d will nt vary by period. Thus 5 period binomial tree bul wi the sae parameters as above would Took ike Figure 1 lscounted atthe sk fee rate. Many students eso eacted by ti, they forget hat esk-newtal probablies are purely ammathematcal concep! and do nt represent the probably of nyting, ; ‘The risk-neutral probability Is purely a mathematical concept and does not represent the 1) probation 1 ueton aks you forthe proba of an event and dws nt ‘qualify the word “probly” with "viak-neutr, tate the true probly. Do mt arse with he riskeneutel probability. 3.2. Replicating portfolio (Originally; he binomial mode! was priced by creating replicating portfolio forthe option, one thathestesame ‘outcomes inal scenarios asthe option. Aswe shall se, no assumption about probability sneeded to construct this portfolio. Tae law of one prc say that wo portfolios lead othe same outcomes in ll scenarios they must have the sume price. na univers with noarbiage, the lw of one price must hold etherwse one could buy the cheaper portfolio and sel the more expensive porio eturns au hat using sk-netral pricing ‘equttalent to pricing with replicating porto hee ow the validity of rk-neutral peng wes proved. ‘Given tha risk-noual pricing is easier eat, thats the method you wil usualy use. However, ets 20, Foro=03: = $0150.61. 173611 d= 1-029 = 0.969050 wae amen Bos nino nsnt rites sr008. one PERIOD Since the option pays of regardless, Is wrth the discounted value ofthe expected pay, whic is the ‘exces othe forward priceathe stock 254°") over the strike price (This works out toe (2548! ~-20)= 5.20, ‘bu you dont need to calculate ths) Since the option Is worth the same for Both volts, the eiference is By formal. andinthiseate Cy (110711) =26= 1.787788, Cy 0. (gh = oonsanoasin pscettiedE Inls=o01+0 Vie o2s2361=001+0 V7 250364~0 Vi = [aaseee7] vie 1825, Fore pu the pleating pontoio ends money, so the answer wl be negate “The upper node ie «= e881 = | 16189 and the lower node ed 77800. The put pays 4007788438) = 12712 at he lower node and nothing a he Upper node since 161845) = 40.64 > 10, Using formula 3.2), ra (etezat) und “ (cises-ae0) ‘making the answer | =36705] 326. Weave the flowing values othe upper and lower nodes: M50. 134906 EXERCISE SOLUTIONS FORLESSON 7 ‘So equate the repeating portfolio athe ler node to: oxis0x0ais7s)+ Be" =0 B=-as0Ko.81073)e 8 =—21.0276 ‘The call premium is AS-+ B=09(90)~21.0276=[ 827, Use equation 3.6), ‘Satements ) and (i were not needed. 8.28, ‘Since the forward price Bde" seater than 80, the opion doesnot pay oat the upper node, Ths Itspice s-*°(a0~o04(1~p). By formula (3.7) for 1~ pina forward ree: 10st2se-* = ¢ 8 —pan125 £329, Lee the numberof shaves ofS in che repliatngportfollo and othe number of hares of Qin the repeating portolio. Let Bbe the amount invested ina risk-free bond, The pay ofthe portfl, rom top bottom, ae 0, 50 and 110. Then 905 F3nay + Be = 40 20a + 1009+ 86" =50 2008+ Be = 10 ‘Sbiratthe second equation ftom the fist and 70 = 10,50 Ao =f, Then from the thitd equation, Bet. 110~200/7 81.4286, Then plugging into the fs equation, 0/7 —B1.4266 Treanor aor st-oso7e 107 +8296" [OE] ‘Alternatively tp, p and p be the rk- neutral probabilities Othe hee outcomes. Then s0pt-+90p$=506% =p 0p} +100p5-+2001-f5")=1006"™ => a0pt+ 1009: Subtracting the fist equation from 3 mes the second equation, angie 260 cis Smoresixat 5. BINOMIAL TREFS STOCK, ONE PERIOD ro BA soos px OEM ans “Te opton porto is worth (a sens +-s(0 05085) + 1.41060) fn," Wecan rep yang ana bond Leta be te munbeof shares of Xt buy and be amount ‘otneatnabont Then sae®824 pet 79 soa? 4 pet 39 ‘Subtracting the second equation fom the ist 2010 bet =70-30(-2)= 130 Then Y0)=40a+b=40(-190010}+ 190 “te eects Cana be done wit rake ODL, Let pb he probabity of Outcome which coresponds tou nour lois y formula 3), snes 59 _ 40e8®—s0 aI=3 sothepriceat Ys + (o4anmi +1 -0499901790) mere ‘Quiz Solutions 3-1. Now the payois 10 the price goes down to 40, Othe price goes up 10.60, We therefore mip the Aiscounted value of 10 ines the probably of 10, or (5365) 82. Now therange fending sok pies ssil20 but hedierence erween the coresponding put pyots— P1501) 2° (05281840) +(1 08281810) “The put premium sthen ~0560) +2860 5360 the sme as the soltion to Quiz 3-1 (QUIZ SOLUTIONS FOR LESSON'3 8 a8. 1. The run stil20 and we sl need e-*© shares to end up wih share tthe end, The puts worth Oif ‘he stock price 560 and ISifit ls 40, ar adifrence of 15.50 (604 ne (-079515)61e"4 Re B= (073515y60}e- = 4200532, The patpeniamls-a7351950)+ 20592 2. By putea parity (85,1) POSS, 1)=se-*—Ke“r 2.7402 —Ys5,1)= 5064 ~ 556-0 19.0099- 52.0176 -3.2077 185,1)=27902+3077 = [6079] 4.35€02900 43.3508 ur tenon 4. BINOMIAL TREES STOCK ONE PERIOD Lesson 4 Binomial Trees—General ‘Reading: Derisatves Market 102-105, exchding "Options on Commodies” on page 394 4.1 Multi-period binomial trees ‘Binomial tes usually have ts of periods. An option spiced by staring atthe end of there and recursively rmovingbeckwarés "BxawLe 4A Fora 6-momh European put option on e stock, you ate give: (@ Thestockpriceis 1s (i) TMhestke price i 160, (i) w=13andd=07, (tw) The continuously compounded rik-re rte is (0) There ate no dividend, The opton is modeled with x2-perad binomial tes, Determine she option premium, Answer: We daw tho toe starting atthe lef and multiplying each number byw and to ge the nest 2 nodes lrtheright Atte right ofthe te, weit stock prices with opdon values below therm in ales ° a wos a ee ae os Hach period of he wee is 3 month. Therisk neu proebllty fan increases v 0525188 We wl pullback rom the x 2 ups and wd (up and down) node onthe right tthe w node. Let Po ‘he put valueat the wu node and Pg 23. be the puvalueat the wd node, fy, the vale at he wmode is -N(ptPax +p") Similar we pl beck fom du —1d and da to 2 0 (as1840 0474s) —1099385 2 009 (9.525106285)+0.474812(065) 261701, sae . “Thon we pullback om nodes w and dt he nal node = 48952510910 99195) +-0.47401250.61701) = [BORE] Fora European option for which no early exercises posil is possible to compute the option premiom without compating the intermediate nodes. The payos are the valves ofthe option tthe ending nodes. The “isbn of probable ofthe ending node isa binomial distribution with parameters m = nthe number of periods, and q= phe risk-neutral probably ofan up movement. Thus we ealulate the expected value of the ending nodes using this distbutton and then discount forthe l perio, Intheabove example, te probabilities ote 3 nodes are | (Joseaueyaezens wa (essen 20 | (2)o=osenny

Вам также может понравиться